PDA

Archiv verlassen und diese Seite im Standarddesign anzeigen : Ist die Dehnung des Raums Physik?


Timm
15.02.14, 16:59
Wenn Andreas Müller hier (http://www.scilogs.de/einsteins-kosmos/dunkle-energie-blo-einbildung/) schreibt

Was dehnt sich?

Das, was expandiert ist der Raum selbst, und - achja, Einstein lehrte das - eigentlich die Raumzeit. ...
könnte man ja nach einem Gedankenexperiment Ausschau halten, das zeigt, daß tatsächlich der Raum selbst sich ausdehnt.
Es gibt Testpartikel, Uhren, Maßstäbe, Lichtpulse ... .
Aber wie könnte so ein Experiment aussehen?
Man hat irgendwo ein Stück Raum, keine störenden Massen in der Nähe und Zeit genug.

Ich
15.02.14, 20:24
Hallo Timm,

meine Antwort kennst du. Aber ich würde mich auch über Rückmeldung von den anderen freuen, wie sie das so sehen.
Kann man Raumexpansion im Prinzip detektieren?
Wenn ja, wie?

JoAx
16.02.14, 01:33
Meine Einstellung ist, dass "Raum" ein Denkkonzept ist. "Man kann in den Raum keine Nägel einschlagen."

Das hier:

Dunkle Energie verhält sich vollkommen anders als normale Materie und sorgt für die Expansion des Universums.

ist schlicht und ergreifend falsch. Das Modell vom expandierenden Universum gab es laaaange vor dem 30/70 ΛCDM-Modell. Die DE sorgt für die Beschleunigung der Expansion.

Ansonsten ist der Artikel halt populärwissenschaftlich, und etwas älter. Das 30/70 ΛCDM "lebt" auch nach dem Plank. :)

Marco Polo
16.02.14, 04:04
Kann man Raumexpansion im Prinzip detektieren?
Wenn ja, wie?

Mit einer kettenartigen Messung von Zweiweg-Lichtlaufzeiten (Radar-Distance)?

Bauhof
16.02.14, 11:18
...könnte man ja nach einem Gedankenexperiment Ausschau halten, das zeigt, daß tatsächlich der Raum selbst sich ausdehnt.
Es gibt Testpartikel, Uhren, Maßstäbe, Lichtpulse ... .
Aber wie könnte so ein Experiment aussehen?
Man hat irgendwo ein Stück Raum, keine störenden Massen in der Nähe und Zeit genug.

Hallo Timm,

darüber habe ich bereits im Jahre 1996 nachgedacht. Hier das Gedankenexperiment:

In einem Gebiet im Universum, der im Umkreis von mehreren Millionen von Lichtjahren materiefrei ist, werden zwei Sonden A und B im Abstand von einem Lichtjahr platziert.

Jede Sonde ist wie folgt bestückt:
(1) Antennen zur Messung der kosmischen Hintergrundstrahlung.
(2) Synchronisierte Atomuhren, die Zeittelegramme erzeugen können.
(3) Sendeanlagen, um Zeittelegramme senden und empfangen zu können.
(4) Echtzeit-Rechner, die sich laufend gegenseitig überwachen.
(5) Steuerdüsen, die in alle Himmelsrichtungen feuern können, um die möglichen Pekulariarbewegungen der Sonden auszugleichen.

Wenn die Sonden platziert sind, wird in den beiden Sonden laufend nach allen Seiten die Rotverschiebung der kosmischen Hintergrundstrahlung gemessen. Diese Messwerte steuern die Steuerdüsen, um die Pekulariarbewegungen der Sonden relativ zur kosmischen Hintergrundstrahlung auf den Wert Null zu bringen.

Wenn beide Sonden relativ zur kosmischen Hintergrundstrahlung ruhen, sendet A nach B ein Zeittelegramm.

Der Echtzeit-Rechner von B vergleicht das Zeittelegramm von A mit dem Zeigerstand seiner eigenen Atomuhr, [1] merkt sich die Differenz und sendet sofort sein eigenes Zeittelegramm nach A.

Sonde A empfängt das Zeittelegramm von B und sendet sofort sein eigenes Zeittelegramm nach B.

Der Echtzeit-Rechner von B vergleicht das Zeittelegramm von A mit dem Zeigerstand seiner eigenen Atomuhr, merkt sich die Differenz und vergleicht die Differenz mit der Differenz, die er sich vor einem Jahr gemerkt hat.

Wenn beide Differenzen gleich groß sind, dann hat sich der Abstand der beiden Sonden nicht vergrößert.

Wenn beide Differenzen nicht gleich groß sind, dann hat sich der Abstand der beiden Sonden vergrößert. Und zwar allein durch die Dehnung des Raumes, denn eine Abstandsvergrößerung durch mögliche Pekulariarbewegungen wurde durch die Messungen der kosmischen Hintergrundstrahlung ausgeschlossen.

M.f.G. Eugen Bauhof

[1] die mit der Atomuhr von A synchron läuft.

Jogi
16.02.14, 12:14
Schönes Experiment.:)

Was würde es am Ergebnis der Zeittelegramvergleiche ändern, wenn nicht der Raum, sondern die Zeit (global) eine Änderung erfahren hätte?

Ich schreibe absichtlich nicht Dehnung, weil ich das Gegenteil erwarte, wenn die beiden Sonden tatsächlich ihren raumartigen Abstand voneinander genau einhalten.

Ein expandierendes Universum "steigt" in meiner Vorstellung aus, bzw. in einem skalaren Grav.-Potential (das sich durch die Expansion "verdünnt") auf, die Zeitdilatation sollte also abnehmen.
Das Problem hierbei ist die fehlende Referenzuhr, die es nicht geben kann, denn sie müßte sich der Expansion entziehen können.

Bauhof
16.02.14, 12:47
Ein expandierendes Universum "steigt" in meiner Vorstellung aus, bzw. in einem skalaren Grav.-Potential (das sich durch die Expansion "verdünnt") auf, die Zeitdilatation sollte also abnehmen.

Hallo Jogi,

1. welches Grav.-Potential?
Es ist doch weit und breit keine gravitierende Massenansammlung vorhanden.

2. Wohin soll ein expandierendes Universum "aus"- oder "aufsteigen"?

M.f.G. Eugen Bauhof

Jogi
16.02.14, 16:00
Hallo Eugen.


1. welches Grav.-Potential?
Es ist doch weit und breit keine gravitierende Massenansammlung vorhanden.
Deshalb "skalares" Potential.

Auch wenn keine baryonische Materie vorhanden ist, so ist das Vakuum doch nicht eine absolute Leere.
Und das, was in diesem Vakuum unsichtbar existent ist, hat, wenn man es nicht direkt selbst als Energie bezeichnen mag, doch ein Energiepotential und damit ein Massenäquivalent.
Eine homogene und isotrope Verteilung dieses Potentials angenommen, kommt man eben zu einem skalaren Potential, das sich durch reale Expansion "verdünnt".


2. Wohin soll ein expandierendes Universum "aus"- oder "aufsteigen"?
Das entspricht der Frage, wohin sich ein Universum denn ausdehnen soll.

Ich weiß es auch nicht, aber wir beobachten global eine Expansion der sichtbaren Materieverteilung im Universum.
So steigen z. B. bei zwei voneinander weg strebenden Galaxien beide jeweils für sich aus dem Potential der anderen auf, und das gilt ja prinzipiell für alles im Universum.

Timm
16.02.14, 16:41
Hallo Eugen,

Wenn beide Differenzen nicht gleich groß sind, dann hat sich der Abstand der beiden Sonden vergrößert. Und zwar allein durch die Dehnung des Raumes, denn eine Abstandsvergrößerung durch mögliche Pekulariarbewegungen wurde durch die Messungen der kosmischen Hintergrundstrahlung ausgeschlossen.

Wenn ich Deine Anordnung richtig verstehe, bedeutet sie, daß sich mitbewegte (-> jeweils in Ruhe zu ihrem CMB) Beobachter gegenseitig rotverschoben sehen und sich das mitteilen. Du hast also quasi die kosmologische Rotverschiebung, bei der Pekuliargeschwindigkeiten kaum noch eine Rolle spielen, experimentell mit großer Genauigkeit nachgestellt.

Ich glaube, es ist naheliegend zu sagen, wenn die Abstände zunehmen (hast Du ja gemessen), muß sich der Raum zwischen den Objekten gedehnt haben (oder zusätzlicher Raum hinzugekommen sein). Aber es tauchen Fragen auf, wie etwa:

Wenn die Dehnung von Raum ein physikalischer Prozess ist, wie soll man sich den vorstellen? Kann man Nichts dehnen?

Sollte ein solcher physikalischer Prozess unabhängig von den verwendeten Koordinaten sein? Mitbewegte Koordinaten bieten sich zwar an, es gibt aber auch andere.

Das Photon wird im lokalen IS bei A emittiert und kommt nachdem der Skalenfaktor zugenommen hat in einem davon verschiedenen lokalen IS bei B rotverschoben an. Wie interpretiert man das?

Gruß, Timm

Bauhof
16.02.14, 17:48
Wenn ich Deine Anordnung richtig verstehe, bedeutet sie, daß sich mitbewegte (-> jeweils in Ruhe zu ihrem CMB) Beobachter gegenseitig rotverschoben sehen und sich das mitteilen. Du hast also quasi die kosmologische Rotverschiebung, bei der Pekuliargeschwindigkeiten kaum noch eine Rolle spielen, experimentell mit großer Genauigkeit nachgestellt.

Ich glaube, es ist naheliegend zu sagen, wenn die Abstände zunehmen (hast Du ja gemessen), muß sich der Raum zwischen den Objekten gedehnt haben (oder zusätzlicher Raum hinzugekommen sein).

Hallo Timm,

um Missverständnisse zu vermeiden, schildere ich das Gedankenexperiment nochmals mit anderen Worten:

Jede der beiden Sonden bringt sich relativ zur Hintergrundstrahlung in Ruhe. Wenn das so ist, dann befinden sich beide Sonden auch relativ zueinander in Ruhe, weil jede Sonde sich relativ zur Hintergrundstrahlung in Ruhe befindet.

Wenn nun die Sonden sich mehrmals die Uhrzeit gegenseitig zusenden und die Radiosignallaufzeit ist immer gleich, dann hat sich der Abstand zwischen den Sonden nicht verändert. Eine Veränderung der Radiosignallaufzeit würde eine Veränderung des Abstandes bedeuten, ganz gleich, wie groß der Abstand tatsächlich ist.

M.f.G. Eugen Bauhof

P.S.
Das das eine riesige Herausforderung an die Messtechnik bedeuten würde, ist wohl klar.

Jogi
16.02.14, 19:02
Eine Veränderung der Radiosignallaufzeit würde eine Veränderung des Abstandes bedeuten, ganz gleich, wie groß der Abstand tatsächlich ist.


...und das wage ich, zu bezweifeln.

Wie Einstein sagte, es handelt sich bei der Expansion nicht um Raum-, sondern um Raumzeitexpansion.
Und weil das so ist, wird die die rein räumliche Abstandsvergrößerung durch eine abnehmende Signallaufzeitdilatation kompensiert.

Timm
16.02.14, 19:19
Hallo Eugen,


Jede der beiden Sonden bringt sich relativ zur Hintergrundstrahlung in Ruhe. Wenn das so ist, dann befinden sich beide Sonden auch relativ zueinander in Ruhe, weil jede Sonde sich relativ zur Hintergrundstrahlung in Ruhe befindet.
Wie meinst Du mit "relativ zueinander in Ruhe"? Ändert sich der raumartige Abstand bei aufeinander folgenden festen Zeitpunkten nicht?

Eine Veränderung der Radiosignallaufzeit würde eine Veränderung des Abstandes bedeuten, ganz gleich, wie groß der Abstand tatsächlich ist.

Nur damit Klarheit herrscht, Du erwartest beim expandierenden Universum zunehmende Laufzeiten, also zunehmende Abstände, oder?

Gruß, Timm

Timm
17.02.14, 09:57
...und das wage ich, zu bezweifeln.

Wie Einstein sagte, es handelt sich bei der Expansion nicht um Raum-, sondern um Raumzeitexpansion.
Und weil das so ist, wird die die rein räumliche Abstandsvergrößerung durch eine abnehmende Signallaufzeitdilatation kompensiert.
Jogi, vielleicht sollten wir uns erst mal einigen, was wir unter Abstand in gekrümmter Raumzeit meinen.

Abstand in mitbewegten Koordinaten: Der ist im dynamischen FRW Modell zeitlich konstant, die Galaxien bewegen sich nicht voneinander fort. Wenn das so ist, sagt Eugen, die Signallaufzeiten aber zunehmen, kann das nur durch eine Dehnung des Raums erklärt werden.

Abstand definiert als "Eigenlänge" (proper distance): Die Länge zwischen 2 Objekten zu einem festen Zeitpunkt, gemessen mit Maßstäben. Diesen Abstand kann man deshalb mit Lichtsignalen nicht messen, er ist ja raumartig.

Welche Abstandsdefinition meinst Du nun mit "räumliche Abstandsvergrößerung "? Und was bedeutet "abnehmende Signallaufzeitdilatation" konkret bezogen auf die Signallaufzeiten zwischen 2 mitbewegten Objekten? Ich will nur Missverständnisse ausschließen.

Gruß, Timm

Timm
17.02.14, 10:18
Mit einer kettenartigen Messung von Zweiweg-Lichtlaufzeiten (Radar-Distance)?
Angenommen, es gelingt eine Abstandsänderung zwischen 2 inertialen Objekten zu detektieren. Vielleicht nicht den exakten Abstand zu festen Zeitpunkten, aber zumindest 'nimmt zu' oder 'nimmt ab'. Welches Kriterium bräuchte man dann, um diesen Befund als Raumdehnung oder als Auseinander-Bewegung, oder als beides kombiniert zu unterscheiden?

Bauhof
17.02.14, 10:26
Wie meinst Du mit "relativ zueinander in Ruhe"? Ändert sich der raumartige Abstand bei aufeinander folgenden festen Zeitpunkten nicht?

Hallo Timm,

ich weiß nicht was du mit "raumartigen" Abstand meinst. Wenn das Universum nicht expandiert, dann ändert sich der Abstand zwischen den beiden Sonden zu keinem Zeitpunkt.

Nur damit Klarheit herrscht, Du erwartest beim expandierenden Universum zunehmende Laufzeiten, also zunehmende Abstände, oder?

Ja, wenn das Universum expandiert, dann erwarte ich mit dem Fortschreiten der Zeit zunehmende Abstände zwischen den beiden Sonden, obwohl sie sich relativ zur kosmischen Hintergrundstrahlung nicht bewegen.

Ich wundere mich ein wenig, denn das von mir vorgestellte Gedankenexperiment basiert doch nur auf die hausbackene Standard-Kosmologie, wie sie z.B. der Physiker Martin Bäker hier (http://scienceblogs.de/hier-wohnen-drachen/2010/09/19/wie-gross-ist-das-beobachtbare-universum/) anschaulich vorstellt.

M.f.G. Eugen Bauhof

Ich
17.02.14, 15:28
Ja, wenn das Universum expandiert, dann erwarte ich mit dem Fortschreiten der Zeit zunehmende Abstände zwischen den beiden Sonden, obwohl sie sich relativ zur kosmischen Hintergrundstrahlung nicht bewegen.

Ich wundere mich ein wenigIch denke Timm fragt nach folgendem: Man könnte ja auch behaupten, die Schwerpunktsysteme des CMB an den beiden Orten seien nicht zueinander in Ruhe. Dann sind auch die darin ruhenden Sonden nicht zueinander in Ruhe, sondern bewegen sich auseinander. Kann man diese beiden Interpretationen mittels eines Experiments unterscheiden?

Oder, anders ausgedrückt: Dass das Universum expandiert ist klar, das sieht man an den Beobachtungen der Dinge darin, inklusive CMB. Als Grund wird gerne "Raumdehnung" genannt. Kann man diese Raumdehnung durch irgendein lokales Experiment von einer schlichten Auseinanderbewegung der im Raum befindlichen Dinge unterscheiden?

Bauhof
17.02.14, 16:36
Oder, anders ausgedrückt: Dass das Universum expandiert ist klar, das sieht man an den Beobachtungen der Dinge darin, inklusive CMB. Als Grund wird gerne "Raumdehnung" genannt. Kann man diese Raumdehnung durch irgendein lokales Experiment von einer schlichten Auseinanderbewegung der im Raum befindlichen Dinge unterscheiden?

Hallo ICH,

ja, ich denke, da gibt es unterschiedliche empirische Konsequenzen. Die zeigt Bernulf Knaitscheider in seinem Buch [1] auf. Er zitiert darin ein bekanntes Werk der Kosmologie, nämlich das von Misner, Ch. / Thorne, K. / Wheeler, J.: Gravitation. Er schreibt auf Seite 226 seines Buches [1] folgendes:

Diese zweite Sichtweise, wo die Galaxien durch das metrische Führungsfeld getragen werden, ist natürlich die relativistische, aber der Unterschied ist nicht nur eine philosophische facon de parier, sondern die beiden Deutungen haben auch unterschiedliche empirische Konsequenzen. Wäre die kosmologische Rotverschiebung tatsächlich ein Geschwindigkeits- und kein Expansionseffekt, so würde der Energiefluss S, der von einer Galaxis der Leuchtkraft L auf der Erde gemessen wird, nicht

S = L / 4πR²(1 + z)², sondern S = L / 4πr²(1 + z)⁴ betragen, siehe
Misner, Ch. / Thorne, K. / Wheeler, J.: Gravitation. San Francisco 1973. (http://www.amazon.de/Gravitation-Physics-Charles-Archibald-Paperback/dp/B00BT04ZN8/ref=sr_1_1?s=books&ie=UTF8&qid=1392653348&sr=1-1)
S. 794, Exercise 29.5

Man muss allerdings bedenken, dass die beiden Deutungen einen Überschneidungsbereich haben, der bis etwa z = 0,1 reicht. Für solche kosmischen Quellen, die auf Entfernungen stehen, die klein gegenüber der Hubble-Länge (dem beobachtbaren Universum) sind, liefern das klassische Doppler-Gesetz und das Expansion-Rotverschiebung-Gesetz quantitativ gleiche Aussagen.

Dies darf jedoch nicht verschleiern, dass der Unterschied begrifflich wesentlich ist. Das sieht man am besten, wenn man extreme Situationen betrachtet. Relativbewegungen von Galaxien durch den Raum sind durch die Lichtgeschwindigkeit c beschränkt. Aufgrund des v = H•D-Gesetzes gibt es in einer endlichen Entfernung vom Beobachter eine sphärische Fläche, auf der die Galaxien mit c vom Beobachter wegstreben.

Deutet man die Rezessionsbewegungen der Galaxien jetzt als Relativbewegung in einem flachen, undynamischen Hintergrundraum, in einer Arena, die nicht am physikalischen Geschehen teilnimmt, so müsste man sich diese Hubble-Sphäre als einen Rand des Universums vorstellen,...

Das heißt, Misner, Thorne und Wheeler kennen offenbar astronomische Beobachtungen, welche einen Rückschluss darauf zulässt, dass die Universum-Expansion kein Geschwindigkeitseffekt, sondern ein Raumdehnungs-Effekt ist.

M.f.G. Eugen Bauhof

[1] Kanitscheider, Bernulf
Kosmologie. (http://www.amazon.de/Kosmologie-Geschichte-Systematik-philosophischer-Perspektive/dp/3150080258/ref=sr_1_1?s=books&ie=UTF8&qid=1392655038&sr=1-1)
Stuttgart 1991
Philipp Reclam jun.
ISBN=3-15-008025-8 (2. Auflage)

Timm
17.02.14, 18:02
Hallo Eugen,

Ja, wenn das Universum expandiert, dann erwarte ich mit dem Fortschreiten der Zeit zunehmende Abstände zwischen den beiden Sonden, obwohl sie sich relativ zur kosmischen Hintergrundstrahlung nicht bewegen.

Ich wundere mich ein wenig, denn das von mir vorgestellte Gedankenexperiment basiert doch nur auf die hausbackene Standard-Kosmologie, wie sie z.B. der Physiker Martin Bäker hier (http://scienceblogs.de/hier-wohnen-drachen/2010/09/19/wie-gross-ist-das-beobachtbare-universum/) anschaulich vorstellt.


Ja, diese Interpretation findet sich häufig. Dein Gedankenexperiment zeigt zunehmende Abstände. Es geht aber um die Frage, ob die Raumdehnung selbst zumindest im Prinzip meßbar ist. Wenn ich irgendeine Substanz dehne, ihr Aggregatzustand spielt keine Rolle, beobachte ich Veränderungen an ihr.

Gruß, Timm

Jogi
17.02.14, 21:10
Hallo Timm.
Jogi, vielleicht sollten wir uns erst mal einigen, was wir unter Abstand in gekrümmter Raumzeit meinen.
:confused:
Eugen geht bei seinem Aufbau doch erklärtermaßen von ungekrümmter Raumzeit aus, oder irre ich mich?


Welche Abstandsdefinition meinst Du nun mit "räumliche Abstandsvergrößerung "?
Raumartig.
Ohne Berücksichtigung raumzeitlicher Effekte.
(Was aber in der Praxis nicht funktionieren wird...)



Und was bedeutet "abnehmende Signallaufzeitdilatation" konkret bezogen auf die Signallaufzeiten zwischen 2 mitbewegten Objekten?
Okay, das ist mißverständlich ausgedrückt.

Ich versuch's nochmal anders:

Eugen's Experiment soll in unserem realen Universum stattfinden, das, wie ich schon sagte, nirgendwo eine absolute Leere aufweist.
Das Quantenvakuum ist immer und überall da und es dilatiert durch seine (skalare, homogen und isotrop wirkende) Gravitation die Zeit und damit auch die Signalgeschwindigkeit im Vergleich zu einem tatsächlich vollkommen leeren Universum.
Natürlich sind in so einem Vergleich auch die Distanzen verkürzt, was zu eben jener von mir angesprochenen Kompensation führt, also einem Nullergebnis des Experiments.

Unser Universum mit seiner abnehmenden globalen Dichte des Vakuums expandiert asymptotisch zu einem "leeren" Universum, aber wie gesagt, die gravitative Dilatation und Längenkontraktion nehmen dabei im Gleichschritt ab, c bleibt immer c.

Ideal als masselos angenommene Referenzobjekte, die sich zum CMB und somit auch zueinander in Ruhe befinden, vergrößern nur aus Sicht des "leeren" Universums ihren Abstand zueinander.

Innerhalb ihres gegenseitigen Wahrnehmungsbereiches ändert sich (theoretisch) nichts.
Eine Zunahme der Rotverschiebung, die das eine Objekt beim anderen beobachtet, wäre nicht zu erwarten.


Gruß Jogi

Bauhof
18.02.14, 14:07
Es geht aber um die Frage, ob die Raumdehnung selbst zumindest im Prinzip meßbar ist. Wenn ich irgendeine Substanz dehne, ihr Aggregatzustand spielt keine Rolle, beobachte ich Veränderungen an ihr.

Hallo Timm,

das ist ein guter Ansatz zum weiteren Verständnis.
Meine Meinung: Die Raumdehnung selbst ist nicht messbar. Der springende Punkt, ist, dass der dreidimensionale Raum allein für sich genommen keine Substanz ist, die sich dehnen kann, gleichgültig, ob sich Materie oder Energie darin befindet.

Ich stelle mir eine Seifenblase vor, die aufgeblasen wird. Auf der Oberfläche der Seifenblase leben (jetzt im Gedankenexperiment) zweidimensionale Wesen, die keine Möglichkeit haben, die dreidimensionale Seifenhaut irgendwie zu erfassen. Für sie existiert nur ein zweidimensionaler Raum. Dass dieser zweidimensionale Raum der Begrenzungsraum einer 3-D-Kugel ist, können sie nicht feststellen.

Nur ein zweidimensionaler Carl Friedrich Gauß könnte mit seinem "hervorragenden Theorem (http://de.wikipedia.org/wiki/Theorema_egregium)" seinen Mitbewohnern Messvorschriften liefern, mit denen sie die eventuelle Krümmung ihrer zweidimensionalen Welt bestimmen könnten, ohne auf eine dritte Dimension Bezug zu nehmen.

Die 2-D-Wesen bemerken nun, dass die Abstände zwischen ihnen immer größer werden, obwohl sich relativ zueinander nicht bewegen. Sie stellen nun die Theorie auf, dass der 2-D-Raum sich im Laufe der Zeit dehnt, was die Abstandsvergrößerung erklären könnte.

In Wirklichkeit kann man aber dem zweidimensionalen Begrenzungsraum die Eigenschaft einer Substanz nicht zusprechen, somit kann er sich auch nicht dehnen. Substanz hat nur die dreidimensionale Seifenhaut, aber die können die 2-D-Wesen nicht wahrnehmen.

Es dehnt sich nur die dreidimensionale Seifenhaut. Nicht aber der zweidimensionale Begrenzungsraum der Seifenblase, weil man dem leeren Raum an sich keine Substanz zusprechen kann.

Somit können die 2-D-Wesen auch keine Veränderung des 2-D-Raumes feststellen, weil er substanzlos ist. Er ist nur ein mathematischer Raum, in dem sich die 2-D-Wesen wie Schatten umherbewegen und der sich im Laufe der Zeit vergrößert.

M.f.G. Eugen Bauhof

Ich
18.02.14, 15:21
Die 2-D-Wesen bemerken nun, dass die Abstände zwischen ihnen immer größer werden, obwohl sich relativ zueinander nicht bewegen.Wie?

Wie stellen sie fest, dass sie sich nicht bewegen? Das ist die Frage.

Den Kanitscheider hatten wir übrigens schon behandelt (http://www.quanten.de/forum/showthread.php5?p=67419), für mich auch abschließend. Im MTW steht nur als Aufgabe, man soll rausfinden, wo der Unterschied herkommt. Und dass die Lösung bei Ellis, 1970 zu finden sei - was ich aber nicht habe.

Marco Polo
18.02.14, 16:20
Wie stellen sie fest, dass sie sich nicht bewegen? Das ist die Frage.

Das ist prinzipiell nicht möglich, behaupte ich jetzt einfach mal ganz kess.

Timm
18.02.14, 17:11
Hallo Jogi,

Eugen geht bei seinem Aufbau doch erklärtermaßen von ungekrümmter Raumzeit aus, oder irre ich mich?
Ja, da irrst Du. In flacher Minkowski-Raumzeit ist jede Frequenzverschiebung zwischen inertialen Beobachtern eine "reine" Dopplerverschiebung. Da gibt's keine Mehrdeutigkeiten und keine Interpretation als Raumdehnung.

Im Folgenden tue ich mir etwas schwer zu verstehen, was Du sagen willst, kann aber auch an mir liegen.
Eugen's Experiment soll in unserem realen Universum stattfinden, das, wie ich schon sagte, nirgendwo eine absolute Leere aufweist.
Das Quantenvakuum ist immer und überall da und es dilatiert durch seine (skalare, homogen und isotrop wirkende) Gravitation die Zeit und damit auch die Signalgeschwindigkeit im Vergleich zu einem tatsächlich vollkommen leeren Universum.
Das Quantenvakuum mit anziehender Gravitation (im Gegensatz zur kosmologischen Konstanten) würde die die Expansion abbremsende Wirkung der Materieenergiedichte weiter verstärken. Es gibt Rechnungen auf Basis der Nullpunktsenergie, aber da ergeben sich irrsinnige Werte, die das Universum zum sofortigen Kollaps brächten. Du nimmst nun eine wohl "schache" Variante an, ist allerdings soz. off-Modell.

Sprichst Du (-> Signalgeschwindigkeit dilatiert) von einer Laufzeitverzögerung im Sinne des Shapiro-Effekts (http://www.spektrum.de/lexikon/physik/shapiro-effekt/13248), nur übertragen von Schwarzschild- auf FRW-Raumzeit?

Natürlich sind in so einem Vergleich auch die Distanzen verkürzt, was zu eben jener von mir angesprochenen Kompensation führt, also einem Nullergebnis des Experiments.
Heißt das bezogen auf Eugen's Experiment, daß während das Universum expandiert die Laufzeiten zwischen den mitbewegten Sonden zeitlich konstant sind?

Unser Universum mit seiner abnehmenden globalen Dichte des Vakuums expandiert asymptotisch zu einem "leeren" Universum, aber wie gesagt, die gravitative Dilatation und Längenkontraktion nehmen dabei im Gleichschritt ab, c bleibt immer c.
Längenkontraktion klingt nach Lorentz-Trafo, wendest Du hier die SRT in der gekrümmten Raumzeit an?

Gruß, Timm

Timm
18.02.14, 17:14
Hallo 'Ich',

meine Antwort kennst du.
ja, etwa seit Du mir im Nachbarkasten Raum zur Detektion geschickt hast. :) Ist schon eine gute Weile her.

Gruß, Timm

Jogi
18.02.14, 18:47
Das Quantenvakuum mit anziehender Gravitation (im Gegensatz zur kosmologischen Konstanten) würde die die Expansion abbremsende Wirkung der Materieenergiedichte weiter verstärken.

Offenbar wirkt die Gravitation im Quantenvakuum jedoch abstossend.

Sorry für OT:
Das liegt in meiner Vorstellung an einer speziellen Eigenschaft der dunklen Materie, die sie grundsätzlich von baryonischer Materie unnterscheidet.
DM wird von Gravitation abgestossen, ihre eigene Gravitationswirkung auf baryonische Materie ist jedoch anziehend, so "zieht" die DM bei ihrer Expansionsbewegung die baryonische Materie mit.


Sprichst Du (-> Signalgeschwindigkeit dilatiert) von einer Laufzeitverzögerung im Sinne des Shapiro-Effekts (http://www.spektrum.de/lexikon/physik/shapiro-effekt/13248), nur übertragen von Schwarzschild- auf FRW-Raumzeit?
Ohne das jetzt im Detail überprüft zu haben, ja.


Heißt das bezogen auf Eugen's Experiment, daß während das Universum expandiert die Laufzeiten zwischen den mitbewegten Sonden zeitlich konstant sind?
Davon gehe ich aus, aber genau das wäre ja mit dem Experiment zu überprüfen.

Denkt man diesen Gedanken mal bezogen auf das, was wir so schon im Universum beobachten können, ergibt sich für mich Folgendes:

Hätten wir vor, na, sagen wir mal 10 Mrd. Jahren die Möglichkeit gehabt, die Rotverschiebung einer weit entfernten Galaxie zu messen, so hätte sich dieser Wert bis heute nicht verändert.
Voraussetzung hierfür wäre jedoch eine ideale, nicht von lokalen Massen beeinflusste Messung (wie sie Eugen vorschwebt).


Längenkontraktion klingt nach Lorentz-Trafo, wendest Du hier die SRT in der gekrümmten Raumzeit an?
Ja.
Im Vergleich zu einem absolut leeren Universum (was sich ja nur denken, aber niemals präparieren lässt...)

Ich
18.02.14, 19:52
Das ist prinzipiell nicht möglich, behaupte ich jetzt einfach mal ganz kess.So sehe ich das auch.

Timm
19.02.14, 14:59
Hallo Eugen,

das ist ein guter Ansatz zum weiteren Verständnis.
Meine Meinung: Die Raumdehnung selbst ist nicht messbar. Der springende Punkt, ist, dass der dreidimensionale Raum allein für sich genommen keine Substanz ist, die sich dehnen kann, gleichgültig, ob sich Materie oder Energie darin befindet.


Da sind wir uns einig. In der ART spielt der Skalenfaktor die Rolle einer dynamischen Größe, aber über eine Dehnung des Raums schweigt sie sich aus.
Meßbar sind Signallaufzeiten, Rotverschiebungen, Winkel unter denen Objekte erscheinen ... . Dann sind wir bei Physik. Die man frei ist zu interpretieren. Für mich bedeutet Expansion zunehmende Abstände. beschäftigen.

Gruß, Timm

Bauhof
20.02.14, 10:18
Da sind wir uns einig. In der ART spielt der Skalenfaktor die Rolle einer dynamischen Größe, aber über eine Dehnung des Raums schweigt sie sich aus.
Meßbar sind Signallaufzeiten, Rotverschiebungen, Winkel unter denen Objekte erscheinen ... . Dann sind wir bei Physik. Die man frei ist zu interpretieren. Für mich bedeutet Expansion zunehmende Abstände. beschäftigen.

Hallo Timm,

ja, da sind wir uns einig, dass der 3-D-Raum nicht etwas Substanzielles ist, das dehnbar ist. Aber sind wir uns auch darüber einig, dass die zunehmenden Abstände bei der Universum-Expansion nicht auf einen Geschwindigkeitseffekt zurückzuführen sind?

M.f.G. Eugen Bauhof

Timm
20.02.14, 17:23
Hallo Eugen,

ja, da sind wir uns einig, dass der 3-D-Raum nicht etwas Substanzielles ist, das dehnbar ist. Aber sind wir uns auch darüber einig, dass die zunehmenden Abstände bei der Universum-Expansion nicht auf einen Geschwindigkeitseffekt zurückzuführen sind?

Ich vermute, daß Du statt "daß" "ob" meinst. Dann sind wir wohl uneins.

Vielleicht hast Du den Nachbar Thread http://www.quantenforum.de/viewtopic.php?f=7&t=618 verfolgt. Die Diskussion ist nicht gerade einfach.
Es gibt zahlreiche teils sich widersprechende Veröffentlichungen zu dem Thema. Aber ein relativer Geschwindigkeitseffekt wird nach meinem Eindruck überwiegend angenommen und eine Dehnung des Raums nicht verteidigt.

Ich belasse es im Moment einfach dabei, daß die Abstände zunehmen, es aber keine eindeutige kinematische Definition gibt. Die gibt's nur in flacher Raumzeit. Die Idee ist, daß beim Übergang von 'lokal noch flach' zu 'fast flach' (bei einem etwas größeren Radius) der lokale Doppler-Effekt immer noch prinzipiell existiert, wenn auch als Folge der Raumzeitkrümmung leicht verändert, wie auch immer das im Detail aussieht. Oder anders, ich kann mir nicht vorstellen, daß es in flacher Raumzeit Relativbewegung gibt, in fast flacher aber schlagartig nicht mehr.

Ich werd' mir heute Abend ein gutes Glas Wein gönnen, auf der Suche nach der Wahrheit.

Gruß, Timm

Bauhof
20.02.14, 17:45
Ich vermute, daß Du statt "daß" "ob" meinst. Dann sind wir wohl uneins.

Hallo Timm,

nein, ich meinte es so, wie ich es schrieb:

"ja, da sind wir uns einig, dass der 3-D-Raum nicht etwas Substanzielles ist."
Das heißt, der 3-D-Raum ist keine Substanz. Ich weiß nicht, wie ich es noch deutlicher ausdrücken soll.

Ich werd' mir heute Abend ein gutes Glas Wein gönnen, auf der Suche nach der Wahrheit.
Im Wein liegt Wahrheit, aber im Bier liegt auch etwas!:)

M.f.G. Eugen Bauhof

P.S.
In der neuen deutschen Rechtschreibung wird statt "daß" nun dass geschrieben

soon
20.02.14, 18:30
Hi,

obwohl sich der Thread nicht in der Plauderecke befindet, versuche ich mal, meine unqualifizierte Meinung zu äußern:

'Raum' als eigenständiges Irgendwas gibt es doch garnicht. Gegenstand der Betrachtung sind immer Gegenstände, oder anders ausgedrückt: Objekte.

Wenn man ganz konsequent die Anschauung beibehält, dass Teile aus Teilen bestehen, dann bestehen Objekte aus kleineren Objekten.
Ab welcher Größenordnung und bis zu welcher Größenordnung diese Anschauung Sinn macht, und ob in 3d, 4d oder mehr, ist eine andere Frage. Aber prinzipiell kann man sagen, dass jedes Objekt innerhalb eines größeren Objekts seinen Platz hat, bzw. dessen Bestandteil ist.
Ganz platt: es gibt keinen leeren Raum, - nicht weil immer etwas drin ist, sondern weil immer ein grösseres Objekt drumherum ist, sozusagen.

Die Frage nach einer Ausdehnung des Raums ist die Frage nach der Ausdehnung eines Objekts.
Vielleicht sollte man aber zunächst versuchen, den Begriff 'Ausdehnung' zu definieren.


[In meiner privaten Physik:D ist 'Raum' sogar nur eine Eigenschaft eines jeden Objekts. Und spannend wird es für mich, in meiner privaten Physik:D, wenn ich versuche Trägheit bzw. Gravitation als Folge der Entstehung von Potentialen durch die Überlagerung der Räume der Objekte zu sehen.]




Zur Ausgangsfrage (da habe ich sicherlich grundlegende Verständnisprobleme), aber als Gedankenexperiment, das zeigt, dass sich der Raum bzw. die Raumzeit ausdehnt, würde ich vorschlagen, die Sterne zu zählen. Die Anzahl der Sterne seit dem Urknall oder sonst einem späteren Ausgangspunkt ist eine diskrete Größe, die stetig wächst. Ein Stern ist dabei der komplette Zyklus Entstehung, Entwicklung, Sterben des Sterns.

LG soon :D

Timm
21.02.14, 08:37
Hallo Eugen,


"ja, da sind wir uns einig, dass der 3-D-Raum nicht etwas Substanzielles ist."

Dein erstes "dass" hatten wir schon abgehakt. Sind wir uns auch beim zweiten, bei dem es um "einen Geschwindigkeitseffekt" geht, einig?

Gruß, Timm

Bauhof
21.02.14, 09:16
Dein erstes "dass" hatten wir schon abgehakt. Sind wir uns auch beim zweiten, bei dem es um "einen Geschwindigkeitseffekt" geht, einig?

Hallo Timm,

die zunehmenden Abstände bei der Universum-Expansion sind nicht auf einen Geschwindigkeitseffekt zurückzuführen.

Kannst du da auch zustimmen?

M.f.G. Eugen Bauhof

Ich
21.02.14, 09:40
die zunehmenden Abstände bei der Universum-Expansion sind nicht auf einen Geschwindigkeitseffekt zurückzuführen.
Womit wir wieder bei der Ausgangsfrage des Threads sind: Kannst du das über ein Experiment belegen oder nicht?

Die Satelliten zeigen, dass das Schwerpunktsystem des lokalen CMB an verschiedenen Orten verschiedene Geschwindigkeit hat.
Kanitscheider zeigt, dass man genau lesen sollte, bevor man zu Schlussfolgerungen springt.

Gibt's noch einen Vorschlag?

Bauhof
21.02.14, 13:43
Gibt's noch einen Vorschlag?

Hallo ICH,

ich erwarte eigentlich von dir einen Literatur-Vorschlag, denn alle Bücher, die ich bisher gelesen habe negieren, dass die Universum-Expansion ein Geschwindigkeitseffekt sein soll.

Kannst du mir wenigstens ein namhaftes deutschsprachiges Buch nennen, in dem die Universum-Expansion als Geschwindigkeitseffekt interpretiert wird? Das würde ich mir kaufen.

M.f.G. Eugen Bauhof

Hawkwind
21.02.14, 13:57
Mir als Kosmolgie- und ART-Laie stellt sich hier die Frage, ob es dabei nicht wieder einmal um Definitionen und Interpretationen geht.

Was wird beobachtet? Die Lichtlaufzeiten zu einem entfernten Objekten nehmen mit der Zeit zu: es entfernt sich von uns. Der Abstand nimmt mit der Zeit zu: diesen Quotienten nennt man in der klassischen Mechanik Geschwindigkeit.

Wenn ich nun zur Raumexpansion kommen will, muss ich ein Modell zurate ziehen, z.B. die ART. Die Beobachtungen alleine bringen mich nach einem Verständnis nicht auf so eine Idee wie Raumexpansion.

Timm
21.02.14, 14:03
die zunehmenden Abstände bei der Universum-Expansion sind nicht auf einen Geschwindigkeitseffekt zurückzuführen.

Kannst du da auch zustimmen?


Nein Eugen, kann ich nicht. Meine Sicht habe ich Dir gestern geschrieben. Aber die ist zugegeben vage.

Gruß, Timm

Bauhof
21.02.14, 14:45
Wenn ich nun zur Raumexpansion kommen will, muss ich ein Modell zurate ziehen, z.B. die ART. Die Beobachtungen alleine bringen mich nach einem Verständnis nicht auf so eine Idee wie Raumexpansion.

Hallo Hawkwind,

das sehe ich auch so. Gemäß Wikipedia (http://de.wikipedia.org/wiki/Expansion_des_Universums) ist der derzeitige Forschungsstand folgender:

Laut der heute gängigsten Theorie ist die kosmologische Rotverschiebung kein Dopplereffekt im eigentlichen Sinne, sondern beruht auf der allgemeinen zeitlichen Zunahme von Abständen im Universum.

Kannst du mir ein deutsprachiges Buch empfehlen, der einen neueren Forschungstand aufzeigt, insbesondere, dass die kosmologische Rotverschiebung auf allen Skalen als Dopplereffekt zu deuten ist?

M.f.G. Eugen Bauhof

Timm
21.02.14, 17:03
Hallo Eugen,

dieses Zitat
Laut der heute gängigsten Theorie ist die kosmologische Rotverschiebung kein Dopplereffekt im eigentlichen Sinne, sondern beruht auf der allgemeinen zeitlichen Zunahme von Abständen im Universum.
finde ich gar nicht so übel.

Kein "eigentlicher" Dopplereffekt (den gibt's nur in der SRT) ja, zunehmende Abstände ja und Raumdehnung geflissentlich nicht erwähnt.

Gruß, Timm

P.S. Meinst Du mit Stand der Forschung die Dehnung des Raums?

Günter Panazée
21.02.14, 17:51
Womit wir wieder bei der Ausgangsfrage des Threads sind: Kannst du das über ein Experiment belegen oder nicht?

Im Experiment misst man auf diese große Entfernungen eine Rotverschiebung (oder keine) - Die Theorie steuert die Hintergründe bei.
Nach der allgemein anerkannten Theorie der Relativität besitzt unser Universum nun einmal keinen Mittelpunkt.
In Privattheorien spricht sicher nichts dagegen eine davon abweichende Annahme zu treffen.

Gonzee

Ich
21.02.14, 19:04
Ich hab' mir hier (http://www.quanten.de/forum/showthread.php5?t=2194)wirklich den Mund fusslig geredet, meinen Standpunkt bis ins Detail genau zu erklären. Und jetzt kommt so was:
Kannst du mir ein deutsprachiges Buch empfehlen, der einen neueren Forschungstand aufzeigt, insbesondere, dass die kosmologische Rotverschiebung auf allen Skalen als Dopplereffekt zu deuten ist?

Das hat echt keinen Sinn, ich bin draußen.

Marco Polo
21.02.14, 21:53
Kannst du mir wenigstens ein namhaftes deutschsprachiges Buch nennen, in dem die Universum-Expansion als Geschwindigkeitseffekt interpretiert wird? Das würde ich mir kaufen.


Hallo Eugen,

zu diesem Thema habe ich im Nachbarforum unlängst geschrieben:

Rebhan [1] schreibt dazu: "Die Deutung der ART-Kosmologie, dass die Zunahme der Galaxienabstände ausschliesslich auf einer Expansion des Raumes beruht, ist auf das System der mitbewegten Koordinaten, für das sie abgeleitet wurde, beschränkt. Auch in der ART-Kosmologie kann man Koordinaten benutzen, in denen sich die Galaxien gegenüber dem Raum bewegen".

[1] Rebhan, Eckhard
Theoretische Physik: Relativitätstheorie und Kosmologie
Springer-Verlag Berlin Heidelberg 2012
ISBN=978-3-8274-2314-6

Grüsse, MP

Marco Polo
21.02.14, 22:27
Ich hab' mir hier (http://www.quanten.de/forum/showthread.php5?t=2194)wirklich den Mund fusslig geredet, meinen Standpunkt bis ins Detail genau zu erklären. Und jetzt kommt so was:


Das hat echt keinen Sinn, ich bin draußen.

Hier reagierst du zu streng, finde ich.

Und überhaupt: Warum machst du dein weiteres Mitwirken in diesem Thread von der Bitte einer einzelnen Person nach einer Buchempfehlung abhängig?

Hier schreiben ja auch noch andere mit, die du damit ungewollt mitbestrafst.

Ausser dir ist zudem kein Kosmologie-Experte anwesend. Insofern ist Eugens Bitte nach Buchtiteln alles andere als unangemessen.

Ist doch schön wenn sich jemand Gedanken macht und nach weiterführender Lektüre fragt. Davor habe ich grossen Respekt. Auch dann, wenn die Frage evtl. bezeugt, dass der Fragensteller etwas nicht verstanden hat und deine bisherigen Bemühungen um Aufklärung möglicherweise für die Katz waren. :)

Grüsse, MP

Günter Panazée
21.02.14, 22:32
"Die Deutung der ART-Kosmologie, dass die Zunahme der Galaxienabstände ausschliesslich auf einer Expansion des Raumes beruht, ist auf das System der mitbewegten Koordinaten, für das sie abgeleitet wurde, beschränkt. Auch in der ART-Kosmologie kann man Koordinaten benutzen, in denen sich die Galaxien gegenüber dem Raum bewegen".

Die GR ist algelmein kovariant - Das ist seit etwas längerem bekannt.

Kannst du mir wenigstens ein namhaftes deutschsprachiges Buch nennen, in dem die Universum-Expansion als Geschwindigkeitseffekt interpretiert wird? Das würde ich mir kaufen.

Das würde mich auch interessieren. Alternative auch Standard-Literatur in Englisch oder Russisch.

Gonzee

Marco Polo
21.02.14, 22:52
Die GR ist algelmein kovariant - Das ist seit etwas längerem bekannt.

Ja selbstverständlich ist sie das. Das gilt nebenbei ja auch für die klassische Mechanik. Ist sie doch invariant gegenüber Galilei-Transformationen.

Das würde mich auch interessieren. Alternative auch Standard-Literatur in Englisch oder Russisch.

Den Buchtitel habe ich weiter oben angegeben.

MP

JoAx
22.02.14, 01:54
Ich versuche mal zusammen zu fassen.

Es geht um die Aussage (ungefähr): dass die Ursache der kosmologischen Rotverschiebung nicht die Geschwindigkeit ist, sondern die Expansion des Raums.

Das Problem bei der Aussage ist, dass:

a) Da in der ART die Geschwindigkeit nicht eindeutig definiert werden kann, kann man diese auch nicht eindeutig als Ursache für irgendetwas ablehnen (wo sie sonst als Ursache in Frage käme).

b) "Expansion des Raumes" ist genau so ein relativistischer Effekt, wie die "kosmologische Rotverschiebung". Die eine Ursache für beide Effekte ("parallel", nicht "nacheinander") ist das expandierende Universum (im 4d-Sinne).

@Günter Panazée

Willkommen bei uns.

Noch ein mal zur Klarstellung - es geht nicht darum, die Expansion des Universums durch Geschwindigkeit zu erklären/begründen. Insofern ist eure mit Eugen Bitte nicht erfüllbar. :)


Grüße, Johann

Günter Panazée
22.02.14, 05:26
Anfänglich spach man von "Nebelflucht".
Ein Kirchenmann extrapolierte dann die Bewegungen in die Vergangenheit und kam zu dem Ergebnis das sich alle Weltlinien in einem Punkt schneiden - Dem Big Bang.
In statischen Koordinaten ist dieser Punkt ausgezeichnet, in mitbewegten nicht.
Deshalb setzte sich schnell die Ansicht durch dass die
Rotverschiebungen auf grossen Skalen auf ein Wachstum des Raums
zurückzuführen sind und man bezeichnete es als kosmologische Rotverschiebung.

Später fand diese Interpretation ihre Bestätigung in verschiedenen
Beobachtungen (z.B. CMB) und ist heute zur Erklärung bestimmter Aspekte des Standardmodells unerlässlich (z.B. Inflation)

Es spricht aber nichts dagegen in Privattheorien von andere Annahmen auszugehen.

Ausser dir ist zudem kein Kosmologie-Experte anwesend.

Ich bin auch Astrologe und habe immer meine Tarot-Karten dabei.

Gonzee

Marco Polo
22.02.14, 07:02
Ich bin auch Astrologe und habe immer meine Tarot-Karten dabei.

Supi. Demnach ein Konjunktions-Fuzzi mit topozentristischer Denke. Na das kann ja was werden. Wir freuen uns dann auf qualifizierte Folgebeiträge.

Und immer schön die Karten trocken halten. ;)

Günter Panazée
22.02.14, 08:34
Der Kirchenmann gelangte auf Grundlage mitbewegter Koordinaten zu der Auffassung das Raum und Zeit auch erst mit dem Urknall entstanden sind.

Gleichzeitig unstrittig ist die allgemeine Kovarianz der GR - wodurch statische und mitbewegte Koordinaten in *ausgezeichneten* Ausschnitten der Raumzeit zu identischen Resultaten führen.

[quote]Das hat echt keinen Sinn[\quote]

Gut erkannt

(Wobei grundsätzlich nichts dagegen spricht Standardmodelle in Frage zustellen^).

Gonzee

Marco Polo
22.02.14, 09:50
Gleichzeitig unstrittig ist die allgemeine Kovarianz der GR

Kovariant unter beliebigen Koordinatentransformationen (Diffeomorphismus-Invarianz) ist meines Wissens die korrekte Beschreibung.

Bauhof
22.02.14, 10:23
dieses Zitat finde ich gar nicht so übel.

Kein "eigentlicher" Dopplereffekt (den gibt's nur in der SRT) ja, zunehmende Abstände ja und Raumdehnung geflissentlich nicht erwähnt.

Gruß, Timm

P.S. Meinst Du mit Stand der Forschung die Dehnung des Raums?

Hallo Timm,

nein, mit Stand der Forschung meinte ich nicht die Dehnung des Raumes, sondern dass die kosmologische Rotverschiebung nicht als Dopplereffekt interpretiert werden darf. Ich denke, das ging doch klar aus dem Zitat hervor.

Seit dreißig Jahren wurde in der Literatur immer wieder betont, dass die kosmologische Rotverschiebung nicht mit dem Dopplereffekt beschrieben werden darf, weil eben kein Geschwindigkeitseffekt bei der Universum-Expansion vorliegt. Beschreibung mit dem Dopplereffekt ist nur auf kleinen Skalen mit guter Näherung zulässig.

Ich weiß, dass "Ich" und du im Gegensatz zu mir Experten auf diesem Gebiet seid. Aber ich hoffe, du hast Verständnis dafür, dass ich trotzdem nach Quellen für eure neue Sichten frage, weil ich dreißig Jahre lang das Gegenteil eurer neuen Sichten gelesen habe.

M.f.G. Eugen Bauhof

Timm
22.02.14, 10:28
Kovariant unter beliebigen Koordinatentransformationen (Diffeomorphismus-Invarianz) ist meines Wissens die korrekte Beschreibung.
Ja.

Unter diesem Gesichtspunkt ist 'Expansion des Universums' mit 'zunehmenden Abständen" hinreichend beschrieben. Geschwindigkeiten sind koordinatenabhängig, die Expansion nicht.

Interessant ist aber finde ich die Frage, ob eine relative Geschwindigkeit zwischen fundamentalen Beobachtern unabhängig von der Wahl der Koordinaten nur ein Teil der kosmologischen Rotverschiebung ist.
Was Sinn macht, denn in der SRT ist die Rotverschiebung global ein Dopplereffekt, in der ART nur lokal. Demnach sollte mit Letzterer ein von der Raumzeitkrümmung abhängiger Anteil der Rotverschiebung hinzu kommen.
Auch dazu sind im Nachbar Thread einige papers aufgeführt.

Marco Polo
22.02.14, 10:50
Interessant ist aber finde ich die Frage, ob eine relative Geschwindigkeit zwischen fundamentalen Beobachtern unabhängig von der Wahl der Koordinaten nur ein Teil der kosmologischen Rotverschiebung ist.

Hervorhebung von mir.

Wie definierst du fundamentale Beobachter?

JoAx
22.02.14, 11:57
Wie definierst du fundamentale Beobachter?

Das sind die, die den CMB exakt isotrop sehen. Wir tuen das aufgrund der Pekuliargeschwindigkeit bsw. nicht.

JoAx
22.02.14, 12:16
In statischen Koordinaten ist dieser Punkt ausgezeichnet, in mitbewegten nicht.
Deshalb setzte sich schnell die Ansicht durch dass die
Rotverschiebungen auf grossen Skalen auf ein Wachstum des Raums
zurückzuführen sind und man bezeichnete es als kosmologische Rotverschiebung.


Und mit der Einführung von 'conformal time'

dτ = dt/R(t); R(t) - Skalenfaktor

ist der Punkt nicht ausgezeichnet, es braucht aber auch keinen Wachstum von Raum, um die Rotverschiebung zu erklären.

http://images.astronet.ru/pubd/2003/11/10/0001194831/map1c.gif

Bauhof
22.02.14, 12:56
Wie definierst du fundamentale Beobachter?

Hallo Marc,

den fundamentalen Beobachter definiert Eckhart Rebhan hier (http://books.google.de/books?id=-iz1Rhv8wLMC&pg=PA429&lpg=PA429&dq=%22Fundamentaler+Beobachter%22&source=bl&ots=p2wqHy38S7&sig=WdhKw_auciDdi7uSrSTUMNVPR8M&hl=de&sa=X&ei=XZoIU-PuHsagtAaCl4DgBA&ved=0CDYQ6AEwAQ#v=onepage&q=%22Fundamentaler%20Beobachter%22&f=false).

M.f.G. Eugen Bauhof

Bauhof
22.02.14, 13:10
Wie definierst du fundamentale Beobachter?

Das sind die, die den CMB exakt isotrop sehen. Wir tuen das aufgrund der Pekuliargeschwindigkeit bsw. nicht.

Hallo Johann,

umgekehrt formuliert sehen fundamentale Beobachter den CMB exakt isotrop, wenn ihre Pekuliargeschwindigkeit Null ist, richtig?

Darf man daraus folgern, dass auch die Relativgeschwindigkeit zwischen zwei Fundamentalbeobachtern Null ist, sofern ihre Pekuliargeschwindigkeit Null ist und beide den CMB exakt isotrop sehen?

M.f.G. Eugen Bauhof

JoAx
22.02.14, 14:37
Darf man daraus folgern, dass auch die Relativgeschwindigkeit zwischen zwei Fundamentalbeobachtern Null ist, sofern ihre Pekuliargeschwindigkeit Null ist und beide den CMB exakt isotrop sehen?


Das Verhältniss zwischen den Fundamentalbeobachtern wird durch Recessionsgeschwindigkeit ausgedrückt. Diese ist nicht Null.

Jetzt wäre interessant zu klären, ob Pekuliargeschwindigkei die Rezessionsgeschwindigkeit "kompensieren" kann.

Grüße

Bauhof
22.02.14, 15:31
Darf man daraus folgern, dass auch die Relativgeschwindigkeit zwischen zwei Fundamentalbeobachtern Null ist, sofern ihre Pekuliargeschwindigkeit Null ist und beide den CMB exakt isotrop sehen?

Das Verhältniss zwischen den Fundamentalbeobachtern wird durch Recessionsgeschwindigkeit ausgedrückt. Diese ist nicht Null.

Jetzt wäre interessant zu klären, ob Pekuliargeschwindigkei die Rezessionsgeschwindigkeit "kompensieren" kann.

Hallo Johann,

von Rezessionsgeschwindigkeit kann man nur dann sprechen, wenn die Abstände zwischen den Fundamentalbeobachtern nicht zu groß ist und daher diese Geschwindigkeit mit dem Dopplereffekt gemessen messen werden kann. Auf größeren Skalen im Universum kann mit dem Dopplereffekt nicht mehr gemessen werden.

Der Unterschied zwischen der Rezessionsgeschwindigkeit und Relativgeschwindigkeit der Fundamentalbeobachter wird hier (http://www.google.de/url?sa=t&rct=j&q=&esrc=s&source=web&cd=3&cad=rja&ved=0CEIQFjAC&url=http%3A%2F%2Fhomepages.physik.uni-muenchen.de%2F~martin.spiessl%2FExpansion%2520des% 2520Raumes.pdf&ei=7roIU4DhJ8btswbVzYGwCQ&usg=AFQjCNGLq8ZohkxLPxJ9rvzp5Wmq7C73FQ&bvm=bv.61725948,d.Yms) im Kapitel "Der Skalenfaktor" gut erklärt:

Diese besteht aus 2 Teilen. Der Erste, die Eigengeschwindigkeit, kommt dadurch zustande, dass sich die Punkte relativ zur Kugeloberfläche bewegen, der Mittelpunktswinkel ändert sich. Der Zweite, die Rezessionsgeschwindigkeit, beschreibt die Ausdehnung des Wegstückes zwischen den Punkten infolge der Ausdehnung der Kugel.

Das ganze lässt sich mit einem Luftballon vergleichen, auf den man Punkte aufzeichnet und dann aufbläst. Die Entfernung der Punkte nimmt proportional zum Radius zu. Da die Punkte fest mit der Luftballonoberfläche verbunden sind, weisen sie natürlich keine Eigengeschwindigkeit auf, es gilt also ϕ = const.

M.f.G. Eugen Bauhof

Timm
22.02.14, 17:46
Hallo Eugen,

nein, mit Stand der Forschung meinte ich nicht die Dehnung des Raumes, sondern dass die kosmologische Rotverschiebung nicht als Dopplereffekt interpretiert werden darf. Ich denke, das ging doch klar aus dem Zitat hervor.

Seit dreißig Jahren wurde in der Literatur immer wieder betont, dass die kosmologische Rotverschiebung nicht mit dem Dopplereffekt beschrieben werden darf, weil eben kein Geschwindigkeitseffekt bei der Universum-Expansion vorliegt. Beschreibung mit dem Dopplereffekt ist nur auf kleinen Skalen mit guter Näherung zulässig.

Ich weiß, dass "Ich" und du im Gegensatz zu mir Experten auf diesem Gebiet seid. Aber ich hoffe, du hast Verständnis dafür, dass ich trotzdem nach Quellen für eure neue Sichten frage, weil ich dreißig Jahre lang das Gegenteil eurer neuen Sichten gelesen habe.

Natürlich habe ich dafür Verständnis. Wie eng sollte man definieren, was man unter Forschung versteht? Viele Publikationen (ich kenne nur englisch sprachige) beschäftigen sich mit der Interpretationen der kosmologischen Rotverschiebung. Mit Fragen wie, gibt es einen kinematischen Anteil, dehnt sich der Raum wirklich, usw. Wie schon erwähnt, findest Du eine kleine Auswahl im Nachbar Forum.
Ich verbinde 'Forschung' mit dem Abgleich Modell/Empirie. Ein Beispiel dafür ist das Lambda-CDM-Modell, dazu gibt's auch Artikel in deutsch. Bei diesem empirisch gut abgesicherten Modell spielt die Frage nach einem Doppler Anteil der kosmologischen Rotverschiebung oder einer Raumdehnung keine Rolle.

Ich habe lange Zeit dasselbe geglaubt wie Du, an eine Dehnung des Raums, und bin erst durch einige Beiträge von 'Ich' im Nachbar Forum mühsam und auf Basis der erwähnten Publikationen von dieser Sichtweise abgekommen. Ein Baustein war auch dieser Artikel von Tamara Davis im Spektrum, den Du glaube ich auch kennst. Ah, doch deutsch, wenigstens einer! Ach, übrigens, das Lesen von ein paar Artikeln macht einen nicht zum Experten, mich jedenfalls nicht.

Gruß, Timm

Timm
22.02.14, 18:07
Das sind die, die den CMB exakt isotrop sehen. Wir tuen das aufgrund der Pekuliargeschwindigkeit bsw. nicht.
Ja, die sog. fundamentalen Beobachter sehen das Universum isotrop und damit auch die kosmologische Rotverschiebung entfernter Galaxien und den CMB.

JoAx
23.02.14, 02:11
von Rezessionsgeschwindigkeit kann man nur dann sprechen, wenn die Abstände zwischen den Fundamentalbeobachtern nicht zu groß ist und daher diese Geschwindigkeit mit dem Dopplereffekt gemessen werden kann.


Ganz im Gegenteil, Eugen.

Ich drücke es mal in der "Sprache des Wachstums des Raumes" und des "Luftballonmodells" aus:

die Rezessionsgeschwindigkeit ist die Änderung des Abstandes einzig und allein aufgrund des "Raumzuwachses"/"Ramschwundes" = Ruhe relativ zur Ballonoberfläche (das ist die Stelle, wo du die Geschwindigkeit Null haben wolltest),
die Pekuliargeschwindigkeit (in der von dir verlinkten Arbeit wird sie "Eigengeschwindigkeit" genannt) ist die Bewegung relativ zur Ballonoberfläche.


Schaue dir noch ein Mal die Stelle um die Formel (14) aus "deinem" Artikel an.

Insbesondere ist wichtig, dass


Der Unterschied zwischen der Rezessionsgeschwindigkeit und Relativgeschwindigkeit der Fundamentalbeobachter wird hier (http://www.google.de/url?sa=t&rct=j&q=&esrc=s&source=web&cd=3&cad=rja&ved=0CEIQFjAC&url=http%3A%2F%2Fhomepages.physik.uni-muenchen.de%2F~martin.spiessl%2FExpansion%2520des% 2520Raumes.pdf&ei=7roIU4DhJ8btswbVzYGwCQ&usg=AFQjCNGLq8ZohkxLPxJ9rvzp5Wmq7C73FQ&bvm=bv.61725948,d.Yms) im Kapitel "Der Skalenfaktor" gut erklärt:


die so genannte "Eigengeschwindigkeit" keine!!! Relativgeschwindigkeit zwischen den Fundamentalbeobachtern ist.

Günter Panazée
23.02.14, 02:25
Ich habe lange Zeit dasselbe geglaubt wie Du, an eine Dehnung des Raums, und bin erst durch einige Beiträge von 'Ich' im Nachbar Forum mühsam und auf Basis der erwähnten Publikationen von dieser Sichtweise abgekommen.

Interessant. Werde konkreter:
Wie lauten die Gründe die Vorstellungen Urknall und Inflation abzulehnen? Was tritt an ihre Stelle?
Was ist mit dem kosmologischen Prinzip? Gilt das noch? Wie?
Wie werden die Charakteristika des CMB alternative erklärt?
Ist das conformal time-Digramm oben falsch?
Wie werden Beschleunigungen bei der Expansion erklärt?
etc.

Solche Fragen muß die Privattheorie detailliert beantworten soll man sie ernst nehmen - Sowas geht nicht:

Ich bin raus

Die Beweispflicht liegt *nicht* beim Standardmodell.
Sonst könnte man gleich das Pendel bemühen.

Im Übrigen bezeichnet man im Standardmodell das normalerweise als Expansion oder Wachstum (Bei einer Dehnung würden keine neuen Beoabachter nachrücken).

Gonzee

JoAx
23.02.14, 02:52
Günter,


Wie lauten die Gründe die Vorstellungen Urknall und Inflation abzulehnen?


Es geht nicht darum, die Vorstellung vom Urknall und Inflation abzulehnen, und schon gar nicht um eine Privattheorie, sondern darum, dass "Raumwachstum" und "Ballonmodell" zumindest didaktisch Fehlgriffe sind. Es geht darum zu ergründen, zu verstehen, warum sie das sind.


Was ist mit dem kosmologischen Prinzip?


Ist das kosmologische Prinzip ein Bestandteil der ART?

(Bei einer Dehnung würden keine neuen Beoabachter nachrücken).


Da haben wir den Salat. Nenne bitte ein Experiment, in dem man diese Nachrückung neuer Beobachter feststellen könnte.

Günter Panazée
23.02.14, 08:20
Es geht nicht darum, die Vorstellung vom Urknall und Inflation abzulehnen, und schon gar nicht um eine Privattheorie, sondern darum, dass "Raumwachstum" und "Ballonmodell" zumindest didaktisch Fehlgriffe sind. Es geht darum zu ergründen, zu verstehen, warum sie das sind.

Es geht um die Interpretation von Redshifts.
Das Standardmodell baut an verschiedenen Stellen auf der Interpretation als kosmologische Rotverschiebung auf: Wenn man Wachstum verneint muß man die entstehenden Lücken nachvollziehbar schließen (siehe die konkret gestellten Fragen).
Kann man das nicht hat man maximal eine Idee formuliert aber seine Hausaufgaben nicht gemacht.

Es spricht überhaupt nichts dagegen anderslautende Privattheorien aufzustellen - Sie dürfen aber nicht anderen weiterhin gültigen Theorien widersprechen.

Ist das kosmologische Prinzip ein Bestandteil der ART?

Selbstverständlich nein wenn man davon absieht dass in der ART kein Koordinatensystem und kein Punkt in der Raumzeit ausgezeichnet ist.

Und es sprichtüberhaupt nichts dagegen anderslautende Privattheorien aufzustellen.
Dann ist das aber nachvollziehbar zu begründen - Insbesondere wenn es mit der Vorstellung, das unser Universum einen Mittelpunkt haben soll, einem Eckpfeiler des Standardmodells widerspricht.
Kann man das nicht hat man maximal eine Idee formuliert aber seine Hausaufgaben nicht gemacht.

Da haben wir den Salat. Nenne bitte ein Experiment, in dem man diese Nachrückung neuer Beobachter feststellen könnte.

Ich schrieb >normalerweise<
Diese Interpretation erlaubt z.B. eine Quantsierung räumlicher Distanzen - Das Standardmodell ist offener für quantengravitative Ansätze (bei einer Dehnung würde man unendlich viele Beobachter aufblättern die vorher einen Abstand unter der Planckgröse komprimiert waren).
Es ist kein Muss - ungewöhnlich. Du verwendest ihn z.B. auch nicht
>Normalerweise< verwendet man einheitliche Begriffe auch um Missverständnisse zu vermeiden.

Was mir aufgefallen ist:
>Ich< konzentrierte sich auch auf die Frage nach einem Experiment.
Mit dir verteidigt jetzt schon der dritte User die Nebelflucht unter Verweigerung der Beantwortung der sich dabei an den Schnittpunkten zu den Standard-Ansichten zwangsläufig ergebenden Fragen.
In einem seriösen Umfeld sind es aber nicht die Standardmodelle die der Beweispflicht nachzukommen haben sondern die Herausforder.

Eher aus Zufall habe ich einmal auf die Forumsmitarbeiter geklickt.
Ich war erstaunt welche Namen ich da gelsen habe.
Deshalb ab sofort viel Spass noch beim Nebelflüchten und vergeßt einfachdie von mir gestellten Fragen.

Gonzee

Bauhof
23.02.14, 09:47
Was mir aufgefallen ist:
>Ich< konzentrierte sich auch auf die Frage nach einem Experiment.
Mit dir verteidigt jetzt schon der dritte User die Nebelflucht unter Verweigerung der Beantwortung der sich dabei an den Schnittpunkten zu den Standard-Ansichten zwangsläufig ergebenden Fragen.
In einem seriösen Umfeld sind es aber nicht die Standardmodelle die der Beweispflicht nachzukommen haben sondern die Herausforder.

Hallo Günter Panazée,

du kannst dir denken, das ich den meisten Ausführungen deines Beitrags zustimme. Insbesondere dazu, bei wem die Beweispflicht liegt.

Nur bei einer deiner Aussagen bin in anderer Meinung: Ich glaube nicht, dass Johann (JoAx) die "Nebelflucht" verteidigt.

Eher aus Zufall habe ich einmal auf die Forumsmitarbeiter geklickt. Ich war erstaunt welche Namen ich da gelsen habe.

Möglicherweise gibt es da Missverständnisse. Bitte sende mir eine persönliche Nachricht, und stelle zu den Forumsmitarbeitern Fragen. Ich denke, ich kann eventuelle Missverständnisse aufklären.

M.f.G. Eugen Bauhof

Günter Panazée
23.02.14, 10:17
Nur bei einer deiner Aussagen bin in anderer Meinung: Ich glaube nicht, dass Johann (JoAx) die "Nebelflucht" verteidigt.

Das weiss ich nicht - Bei ihm habe ich auch nur einen Anfangsverdacht:
Er beantwortet nicht die gestellten Fragen
Er fordert nicht die Beantwortung der gestellten Fragen
Der letzte Beitrag von ihm kann man auch als ersten Versuch einer Beweislastumkehr deuten

Ich kann mich irren - Er möchte seinen Standpunkt verdeutlichen.

Möglicherweise gibt es da Missverständnisse. Bitte sende mir eine persönliche Nachricht, und stelle zu den Forumsmitarbeitern Fragen. Ich denke, ich kann eventuelle Missverständnisse aufklären.

Ich mag keine Geheimsiskrämerei: Zwei (wenn nicht gar drei) Forumsmitarbeiter befürworten eine Privattheorie statt eines etablierten und anerkannten Bestandteils des Standardmodells.

Gleichzeitig unstrittig ist die allgemeine Kovarianz der GR - wodurch statische und mitbewegte Koordinaten in *ausgezeichneten* Ausschnitten der Raumzeit zu identischen Resultaten führen.

Wenn sie ihrer damit verbundenen Beweispflicht (Es hat Auswirkungen auf das Standardmodell wenn man des soeben Zitierten die auf die gesamte Raumzeit ausweitet - Diese sind zu prüfen) nicht nachkommen aber trotzdem weiter ihre Thesen vertreten sind sie keine seriösen Diskussionspartner - So einfach sehe ich das.

Gonzee

Timm
23.02.14, 10:18
Interessant. Werde konkreter:
Wie lauten die Gründe die Vorstellungen Urknall und Inflation abzulehnen? Was tritt an ihre Stelle?
Was ist mit dem kosmologischen Prinzip? Gilt das noch? Wie?
Wie werden die Charakteristika des CMB alternative erklärt?
Ist das conformal time-Digramm oben falsch?
Wie werden Beschleunigungen bei der Expansion erklärt?
etc.

Solche Fragen muß die Privattheorie detailliert beantworten soll man sie ernst nehmen -
Offenbar ist Dir aus dem Verlauf des Threads nicht klar geworden, worum es geht. Niemand stellt das Lambda-CDM-Modell infrage, s. die Post aus der Du zitierst. Auch scheint Dir die Unterscheidung Theorie/Interpretation nicht geläufig zu sein. Falls Du Dich tatsächlich für das Thema interessierst, wären die ersten 3 papers ein Einstieg.

http://arxiv.org/abs/0808.1081 (Bunn&Hogg, "The kinematic origin of the cosmological redshift")
http://arxiv.org/abs/0809.4573 (Peacock, "A diatribe on expanding space")
http://arxiv.org/pdf/0707.0380v1.pdf Expanding Space: the Root of all Evil?

http://arxiv.org/abs/gr-qc/0506032 (Bolos, "Intrinsic definitions of "relative velocity" in general relativity")
http://arxiv.org/PS_cache/arxiv/pdf/091 ... 3536v3.pdf The kinematic component of the cosmological redshift, Michał J. Chodorowski⋆
http://www.mso.anu.edu.au/~charley/pape ... aver04.pdf Expanding Confusion: ..., Davis&Lineweaver
, Matthew J. Francis1,4, Luke A. Barnes1,2, J. Berian James1,3 & Geraint F. Lewis1
http://arxiv.org/pdf/1111.6704.pdf Interpretations of Cosmological Spectral Shifts, Dag Østvang
http://www.ptep-online.com/index_files/ ... -09-06.PDF Cosmological Redshift Interpreted as Gravitational Redshift, Potter&Preston†
http://arxiv.org/abs/1010.0588 (Klein&Randles, "Fermi coordinates,
simultaneity, and expanding space in Robertson-Walker cosmologies")

Günter Panazée
23.02.14, 10:26
Offenbar ist Dir aus dem Verlauf des Threads nicht klar geworden, worum es geht. Niemand stellt das Lambda-CDM-Modell infrage, s. die Post aus der Du zitierst. Auch scheint Dir die Unterscheidung Theorie/Interpretation nicht geläufig zu sein. Falls Du Dich tatsächlich für das Thema interessierst, wären die ersten 3 papers ein Einstieg.

http://arxiv.org/abs/0808.1081 (Bunn&Hogg, "The kinematic origin of the cosmological redshift")
http://arxiv.org/abs/0809.4573 (Peacock, "A diatribe on expanding space")
http://arxiv.org/pdf/0707.0380v1.pdf Expanding Space: the Root of all Evil?

http://arxiv.org/abs/gr-qc/0506032 (Bolos, "Intrinsic definitions of "relative velocity" in general relativity")
http://arxiv.org/PS_cache/arxiv/pdf/091 ... 3536v3.pdf The kinematic component of the cosmological redshift, Michał J. Chodorowski⋆
http://www.mso.anu.edu.au/~charley/pape ... aver04.pdf Expanding Confusion: ..., Davis&Lineweaver
, Matthew J. Francis1,4, Luke A. Barnes1,2, J. Berian James1,3 & Geraint F. Lewis1
http://arxiv.org/pdf/1111.6704.pdf Interpretations of Cosmological Spectral Shifts, Dag Østvang
http://www.ptep-online.com/index_files/ ... -09-06.PDF Cosmological Redshift Interpreted as Gravitational Redshift, Potter&Preston†
http://arxiv.org/abs/1010.0588 (Klein&Randles, "Fermi coordinates,
simultaneity, and expanding space in Robertson-Walker cosmologies")

Das ist alles soweit berechtigt (und bereits im Standardmodell enthalten - Insofern überhaupt nichts "Neues") sofern das beachtet wird:
Gleichzeitig unstrittig ist die allgemeine Kovarianz der GR - wodurch statische und mitbewegte Koordinaten in *ausgezeichneten* Ausschnitten der Raumzeit zu identischen Resultaten führen.

Mit "werde konkreter" meinte ich z.B.

Zwei Beobachter entfernen sich durch die Expansion voneinander.
Beide ruhen jeweils zum CMB.

Erkläre mir bitte den Sachverhalt unter Annahme einer Galaxienflucht statt Raumexpansion.

Gonzee

Bauhof
23.02.14, 11:38
Ich mag keine Geheimsiskrämerei: Zwei (wenn nicht gar drei) Forumsmitarbeiter befürworten eine Privattheorie statt eines etablierten und anerkannten Bestandteils des Standardmodells.

Hallo Günter Panazée,

da höre ich Vorbehalte gegenüber dem Forum als Ganzes heraus.
Das einzige, was man dem Thread-Ersteller anlasten könnte ist, dass er seinen Erstbeitrag in das Unterforum für Standard-Physik eingestellt hat.

Könntest du deine etwaigen Vorbehalte begraben, wenn ich Timm bitten würde, seinen Thread in das Unterforum "Theorien jenseits der Standard-Physik" verschieben zu lassen?

M.f.G. Eugen Bauhof

Günter Panazée
23.02.14, 12:06
da höre ich Vorbehalte gegenüber dem Forum als Ganzes heraus.
Das einzige, was man dem Thread-Ersteller anlasten könnte ist, dass er seinen Erstbeitrag in das Unterforum für Standard-Physik eingestellt hat.

Könntest du deine etwaigen Vorbehalte begraben, wenn ich Timm bitten würde, seinen Thread in das Unterforum "Theorien jenseits der Standard-Physik" verschieben zu lassen?

Da möchte die Kirche im Dorf lassen - Mir ist etwas aufgefallen, ich habe mir noch keine abschließende Meinung dazu gebildet: Es kann etwas dran sein oder auch nichts. Das wird man sehen.
Ein Verschieben ändert daran auch nichts.
Es beeinflusst höchstens meine Entscheidung ob ich weiter posten möchte oder nicht - Das ist aber meine persönliche Entscheidung.
Du bist mir z.B. in der diskutierten Sache mit angemessen kritischen Nachfragen aufgefallen - Das zeigt das man wegen eines Einzelnen nicht ein gesamtes Forum über einen Kamm scheren kann.

Gonzee

Günter Panazée
23.02.14, 12:14
Mit "werde konkreter" meinte ich z.B.

Die Expansion zeigt sich - nicht wie anfänglich angenommen konstant sondern - beschleunigt.
Erkläre bitte den Sachverhalt unter Annahme einer Nebelflucht statt Raumexpansion.
(z.B. Treten bei dieser Beschleunigung Trägheitskräfte auf? Falls ja: Wo / Wie deckt sich das mit den Beobachtungen? Falls nein: Warum nicht / Wie funktioniert das? ....)

etc.

*Alle* Aspekte, die das aktuelle Standardmodell über ein Wachstum des Raums nachvollziehbar erklärt (oder wo auch nur Bezug darauf genommen wird), müssen von einer neuen Theorie der Nebelflucht in der Mindestanforderung geprüft und schlüssig beantwortet werden.

Wenn Du das nicht kannst aber trotzdem an deiner These festhalten solltest halte ich eine weitere Diskussion für sinnfrei:
Da wende ich mich dann lieber wieder in meinem stillen Kämmerlein meinen Tarot-Karten zu.

Gonzee

JoAx
23.02.14, 13:03
Es geht um die Interpretation von Redshifts.


Ich weiss, doch, etwas mehr um die Geschichte des Disputs, als du, Günter. Und ich sage - darum geht es nicht.


Das Standardmodell baut an verschiedenen Stellen auf der Interpretation als kosmologische Rotverschiebung auf:


Das Standardmodell baut vor allem auf der ART auf (was ihren kosmologischen "Anteil" betrifft), und nicht auf irgendwelchen "Interpretationen". Auch in der ART ist die Raumzeit ein absolutes "Ding" (vierdimensionale Mannigfaltigkeit). Wenn die Weltlinien der Beobachter auseinander laufen, dann vergrößert sich der Abstand zwischen diesen. Ansonsten tut da nichts "wachsen".


Wenn man Wachstum verneint muß man die entstehenden Lücken nachvollziehbar schließen (siehe die konkret gestellten Fragen).


Welche Lücken? Du gehst davon aus, dass wenn man "Wachstum" verneint, dass man damit automatisch von der Bewegung vor dem Hintergrund der Minkowski-Raumzeit ausgeht. Wie kommst du darauf?


Es spricht überhaupt nichts dagegen anderslautende Privattheorien aufzustellen - Sie dürfen aber nicht anderen weiterhin gültigen Theorien widersprechen.


Noch ein Mal - es geht nicht darum anders lautende Privattheorie aufzustellen, sondern darum, FLRW zu verstehen.


Selbstverständlich nein wenn man davon absieht dass in der ART kein Koordinatensystem und kein Punkt in der Raumzeit ausgezeichnet ist.


Und wenn man davon nicht absieht?


Insbesondere wenn es mit der Vorstellung, das unser Universum einen Mittelpunkt haben soll,


Zitiere bitte, wo das behauptet wurde.


Diese Interpretation erlaubt z.B. eine Quantsierung räumlicher Distanzen - Das Standardmodell ist offener für quantengravitative Ansätze


Ach so. Du meinst also, dass die Quantengravitation durch Quantisierung räumlicher Abstände erreicht werden kann? Und dass das Standardmodell dem quasi vorgreift? Hmmm, ich dachte, es gibt noch nichts zählbares aus der "Ecke".

Ich schlage vor, auf dem Boden der Tatsachen zu bleiben, und dem Standardmodell nichts auflasten, was es nicht tragen könnte.

Es ist "Raumwachstum", der im besten Falle eine Idee ist, in der Tat aber wohl eher nicht mehr, als eine blöde, mechanistische Analogie. Ich kann mir denken, warum "man" diese eingeführt hat, bin schließlich auch mit ihr aufgewachsen - um der naiven Vorstellung, beim Big Bang würde es sich um eine gewöhnliche Explosion im bereits existierenden Raum handeln, entgegen zu wirken. Ja, dafür ist es gut. Aber damit ist die Belastbarkeit dieser Analogie, dieses Bildchens auch erreicht. Für mehr taugt sie nicht.


Mit dir verteidigt jetzt schon der dritte User die Nebelflucht


Wenn du das denkst, dann hast du den Punkt, um den es geht, noch nicht erfasst. Hier zwei Themen, die du vlt. und bitte durchlesen könntest, bevor du dich weiter einbringst:

Расширение пространства (http://dxdy.ru/topic61477.html)
Расширение Вселенной с ускорением (http://dxdy.ru/topic44029.html)

Nur damit wir nicht weiter aneinander vorbei reden.


Eher aus Zufall habe ich einmal auf die Forumsmitarbeiter geklickt.
Ich war erstaunt welche Namen ich da gelsen habe.


Was stört dich denn? Dass Leute zu lernen versuchen?

JoAx
23.02.14, 13:37
Die Expansion zeigt sich - nicht wie anfänglich angenommen konstant sondern - beschleunigt.
Erkläre bitte den Sachverhalt unter Annahme einer Nebelflucht statt Raumexpansion.


Also, bitte! Wir wollen doch nicht Expansion und beschleunigte Expansion in einen Topf schmeißen!


(z.B. Treten bei dieser Beschleunigung Trägheitskräfte auf? Falls ja: Wo / Wie deckt sich das mit den Beobachtungen? Falls nein: Warum nicht / Wie funktioniert das? ....)


Nein, es treten keine Trägheitskräfte auf. Funktionieren tut es genau, wie die "gewöhnliche Gravitation", nur in entgegengesetzte Richtung. Willst du jetzt die "anziehende Wirkung" der gewöhnlichen Gravitation über "Raumschwund" erklären und behaupten, dass es Standardmodell wäre?

Das wäre ein Ding!

Timm
23.02.14, 14:09
Die Expansion zeigt sich - nicht wie anfänglich angenommen konstant sondern - beschleunigt.
Erkläre bitte den Sachverhalt unter Annahme einer Nebelflucht statt Raumexpansion.
(z.B. Treten bei dieser Beschleunigung Trägheitskräfte auf? Falls ja: Wo / Wie deckt sich das mit den Beobachtungen? Falls nein: Warum nicht / Wie funktioniert das? ....)

etc.

*Alle* Aspekte, die das aktuelle Standardmodell über ein Wachstum des Raums nachvollziehbar erklärt (oder wo auch nur Bezug darauf genommen wird), müssen von einer neuen Theorie der Nebelflucht in der Mindestanforderung geprüft und schlüssig beantwortet werden.


Die Friedmann Gleichungen beschreiben die zeitliche Entwicklung des Skalenfaktors a und damit sich verändernde Abstände, mehr nicht. Nun kannst Du nachsehen unter welchen Bedingungen ä > 0 ist. Was Du fälschlich mit dem Auftreten von Trägheitskräften zu verknüpfen scheinst.

Solange Du 'Interpretation' mit 'neue Theorie' verwechselst, macht eine Diskussion wenig Sinn. Schau Dir die ersten Publikationen an und überdenke Deinen Standpunkt. Peacock ist übrigens ein bekannter Kosmologe und Verfasser von "Cosmological Physics".

Günter Panazée
23.02.14, 14:28
Ich weiss, doch, etwas mehr um die Geschichte des Disputs, als du, Günter. Und ich sage - darum geht es nicht.

Was willst du mir mitteilen?

Das Standardmodell baut vor allem auf der ART auf (was ihren kosmologischen "Anteil" betrifft), und nicht auf irgendwelchen "Interpretationen". Auch in der ART ist die Raumzeit ein absolutes "Ding" (vierdimensionale Mannigfaltigkeit). Wenn die Weltlinien der Beobachter auseinander laufen, dann vergrößert sich der Abstand zwischen diesen. Ansonsten tut da nichts "wachsen".

Demnach verneinst Du den Urknall (insbesondere die Inflation) des Standardmodells. Verstanden.

Welche Lücken? Du gehst davon aus, dass wenn man "Wachstum" verneint, dass man damit automatisch von der Bewegung vor dem Hintergrund der Minkowski-Raumzeit ausgeht. Wie kommst du darauf?

Hat Timm geschrieben: Er hat sich vom Standardmodell abgewendet.

Noch ein Mal - es geht nicht darum anders lautende Privattheorie aufzustellen, sondern darum, FLRW zu verstehen.

Das liest sich aber anders.

Und wenn man davon nicht absieht?

In statischen Koordinaten wäre der Ort des Urknalls ein exakter Punkt in der Raumzeit.

Zitiere bitte, wo das behauptet wurde.

Ich bin so frei dass aus der Verwendung statischer Koordinaten abzuleiten gerade wenn keine Aussage zum Umgang mit dem Big Bang getroffen wird: Die Annahme eines Big Bangs ist schließlich der Grund weshalb man ein Wachstum des Raums annimmt.

Ach so. Du meinst also, dass die Quantengravitation durch Quantisierung räumlicher Abstände erreicht werden kann? Und dass das Standardmodell dem quasi vorgreift? Hmmm, ich dachte, es gibt noch nichts zählbares aus der "Ecke".

Korrekt. Es war ein Test was ich von Dir halten soll - Etwas besseres fiel mir nicht ein.
Eine Dehnung würde bedeuten das Raum und Zeit bereits vor dem Urknall Bestand hatten und durch diesen lediglich entfaltet wurde.
Tatsächlich geht das Standardmodell aber von der Entstehung der Raumzeit mit dem Urknall aus - Deshalb wächst im Standardmodell der Raum.

Ich schlage vor, auf dem Boden der Tatsachen zu bleiben, und dem Standardmodell nichts auflasten, was es nicht tragen könnte.

Ich zähle - wenn ich das Nachfolgende dann noch auf die Schnelle überfliege (Ausflüchte oder für mich unverständliches) - damit *drei* Forumsmitarbeiter die berechtige Fragen unbeantwortet lassen aber dem Standardmodell widersprechen.

Ich wende mich deshalb jetzt wider meinen Karten zu. Das ist seriöser.

Gonzee

Bauhof
23.02.14, 14:32
Ich drücke es mal in der "Sprache des Wachstums des Raumes" und des "Luftballonmodells" aus:

die Rezessionsgeschwindigkeit ist die Änderung des Abstandes einzig und allein aufgrund des "Raumzuwachses"/"Ramschwundes" = Ruhe relativ zur Ballonoberfläche (das ist die Stelle, wo du die Geschwindigkeit Null haben wolltest),
die Pekuliargeschwindigkeit (in der von dir verlinkten Arbeit wird sie "Eigengeschwindigkeit" genannt) ist die Bewegung relativ zur Ballonoberfläche.


Hallo Johann,

einverstanden, so sehe ich diese Dinge auch.
Ich habe mich nur daran gestört, dass man die Änderung des Abstandes aufgrund des "Raumzuwachses" Rezessionsgeschwindigkeit nennt. Ein Relikt aus der Vergangenheit. Denn es ist keine Geschwindigkeit im gewöhnlichen Sinne. Aber Ok, damit kann ich leben.

Schaue dir noch ein Mal die Stelle um die Formel (14) aus "deinem" Artikel an. Insbesondere ist wichtig, dass

die so genannte "Eigengeschwindigkeit" keine!!! Relativgeschwindigkeit zwischen den Fundamentalbeobachtern ist.

Die Relativgeschwindigkeit zwischen den Fundamentalbeobachtern ist also Null, sofern die Fundamentalbeobachter zum CMB ruhen, richtig? Was ist dann die "Eigengeschwindigkeit" der Fundamentalbeobachter?

M.f.G. Eugen Bauhof

Bauhof
23.02.14, 14:51
Ich zähle - wenn ich das Nachfolgende dann noch auf die Schnelle überfliege (Ausflüchte oder für mich unverständliches) - damit *drei* Forumsmitarbeiter die berechtige Fragen unbeantwortet lassen aber dem Standardmodell widersprechen.

Ich wende mich deshalb jetzt wider meinen Karten zu. Das ist seriöser.

Hallo Günter Panazée,

ich bin fest davon überzeugt, dass du etwas vorschnell urteilst.
Die User ICH, Timm und auch Johann sind langjährige seriöse Foren-Mitglieder, die ich sehr schätze. Nur diese eine Mal hat der User ICH etwas daneben gegriffen, indem er den Thread verlies, nur weil ich Quellen in deutscher Sprache verlangt habe.

Ich stimme mit ihnen fast immer überein, nur diesmal heben wir sehr unterschiedliche Meinungen. Das darf doch mal vorkommen, dazu ist ein Forum doch da, um unterschiedliche Meinungen auszutauschen.

Bitte überdenke deine Rückkehr zu deinen "Karten" noch mal.

M.f.G. Eugen Bauhof

JoAx
23.02.14, 16:20
Was willst du mir mitteilen?


Dass so lange du dich auf "Crackpot-Safari" befindest, du hier tatsächlich fehl am Platz bist.

Timm
23.02.14, 18:07
Ich stimme mit ihnen fast immer überein, nur diesmal heben wir sehr unterschiedliche Meinungen. Das darf doch mal vorkommen, dazu ist ein Forum doch da, um unterschiedliche Meinungen auszutauschen.

Na klar, Eugen. Ich weiß aber gar nicht, ob wir wirklich so weit auseinander liegen.

Ich begnüge mich mit der Friedmann Dynamik, die die Abhängigkeit der zeitlichen Entwicklung des Skalenfaktors und damit der Distanzen zwischen fundamentalen Beobachtern von Energiedichte und Druck beschreibt. Expansion bedeutet zunehmende Distanzen.

Nun beginnt ab hier für mich die Interpretation der zunehmenden Distanzen und damit der kosmologischen Rotverschiebung. Da gibt es alternativ zur Dehnung des Raums eine nennen wir's mal an die Krümmung angepasste Dopplerverschiebung, einen Mix aus Dopplerverschiebung und Gravitation und anderes mehr. Diese Autoren, s. die Liste, haben unterschiedliche Sichtweisen, die sie natürlich jeweils begründen. Experimentell verifizierbar ist keine. Einig sind sie sich, daß das Beharren auf der weitverbreiteten Deutung der zunehmenden Abstände als Dehnung des Raums trügerisch ist.

Nun unterscheiden wir uns wohl dadurch, daß Du außer der Dehnung des Raums keine andere Interpretation gelten läßt.

Gruß, Timm

Günter Panazée
23.02.14, 19:16
ich bin fest davon überzeugt, dass du etwas vorschnell urteilst.

Kann sein, Eugen - Ich urteile auf Basis dessen was ich von ihnen lese.

Die User ICH, Timm und auch Johann sind langjährige seriöse Foren-Mitglieder, die ich sehr schätze.

Ich lese mir das durch was deine Freunde aktuell schreiben:

Es ist "Raumwachstum", der im besten Falle eine Idee ist, in der Tat aber wohl eher nicht mehr, als eine blöde, mechanistische Analogie. Ich kann mir denken, warum "man" diese eingeführt hat, bin schließlich auch mit ihr aufgewachsen - um der naiven Vorstellung, beim Big Bang würde es sich um eine gewöhnliche Explosion im bereits existierenden Raum handeln, entgegen zu wirken. Ja, dafür ist es gut. Aber damit ist die Belastbarkeit dieser Analogie, dieses Bildchens auch erreicht. Für mehr taugt sie nicht.

Ich habe lange Zeit dasselbe geglaubt wie Du, an eine Dehnung des Raums, und bin erst durch einige Beiträge von 'Ich' im Nachbar Forum mühsam und auf Basis der erwähnten Publikationen von dieser Sichtweise abgekommen.

<Funkstille>


Beim einen ist es die Raumexpansion, die er ablehnt, der andere mag das Relativitätsprinzip nicht, der dritte widerspricht der relativistischen Geschwindigkeitsaddition u.s.w. u.s.w.

Dabei ist die Motivation im Prinzip immer dieselbe: Ihnen gefällt etwas nicht - Sie verstehen es nicht - Sie lehnen es ab.

Du kannst das gerne bei deinen Freunden überprüfen:

Niemand stellt das Lambda-CDM-Modell infrage, s. die Post aus der Du zitierst.
Es geht nicht darum, die Vorstellung vom Urknall und Inflation abzulehnen
<Funkstille>

Der Urknall baut auf der Expansion des Raums auf und *nicht* auf der Nebelflucht - Wenn man die Expansion ablehnt muß man eine alternative Erklärung an Stelle des Urknalls anbieten (Und es existieren Szenarien ohne Anfangssingularität - so ist es nicht; aber Urknall und Nebelflucht vertragen sich nicht miteinander).

Hinsichtlich des CMB stellst Dun schon die richtigen Fragen.

Wenn Dir deine Freunde nicht wenigstens diese beiden Aspekte ansatzweise schlüssig mit ihrer Privattheorie erklären können - wovon ich ausgehe denn sonst hätten sie schon längst ein Wort dazu verloren - dann sind sie nicht besser oder schlechter als jeder andere Neunmalkluge, der meint, dass an den physikalischen Fakultäten schon seit Jahrzehnten sowieso nur Idioten am Werk sind die einfach nicht die einzig richtige Wahrheit erkennen können oder wollen: Diese Wahrheit ist natürlich ihre.

Und auf unbequeme Fragen reagieren sie dann immer allergisch - Das ist auch immer das Gleiche.

Bitte überdenke deine Rückkehr zu deinen "Karten" noch mal.

Ich diskutiere mit solchen Leuten nicht mehr, Eugen - I don't want to feed them ....
Vielleichtspäter wieder.

Eines interessiert mich allerdings noch:

Willst du jetzt die "anziehende Wirkung" der gewöhnlichen Gravitation über "Raumschwund" erklären

Wie kommst du in Bezug zur Expansion darauf oder woher hast du das? (Irgendwo aufgeschnappt?)

Gonzee

Ich
23.02.14, 20:09
Hallo Eugen,

ich denke, ich habe zwei Beiträge von dir durcheinandergeworfen.
Hallo ICH,

ich erwarte eigentlich von dir einen Literatur-Vorschlag, denn alle Bücher, die ich bisher gelesen habe negieren, dass die Universum-Expansion ein Geschwindigkeitseffekt sein soll.

Kannst du mir wenigstens ein namhaftes deutschsprachiges Buch nennen, in dem die Universum-Expansion als Geschwindigkeitseffekt interpretiert wird? Das würde ich mir kaufen.
In diesem Thread ging es um die Frage, ob man durch Messungen Raumexpansion von Auseinanderbewegung unterscheiden kann. Die Beweislast dafür liegt natürlich bei demjenigen, der die Frage mit "ja" beantwortet. Es ist einfach das entsprechende Experiment zu nennen.


Kannst du mir ein deutsprachiges Buch empfehlen, der einen neueren Forschungstand aufzeigt, insbesondere, dass die kosmologische Rotverschiebung auf allen Skalen als Dopplereffekt zu deuten ist?
Ich habe diesen Beitrag (vielleicht fälschlicherweise) auch noch auf mich bezogen.
Dazu ist folgendes zu sagen: Im anderen Thread sagtest du, für dich sind in dieser Diskussion weder Begründungen durch Mathematik noch durch Verweis auf englische Fachliteratur zugänglich. Deshalb haben wir uns geeinigt, dass du meiner Sichtweise gegenüber skeptisch bleibst, zumindest bis ich deutschsprachige Quellen aufweisen kann. Das ist unter den Umständen absolut vernünftig und ok für mich.
Das schöne ist, dass Marco Polo mittlerweile mit Rebhan tatsächlich eine Quelle aufgetan hat, die meine Sichtweise ausdrückt. (Danke nochmal dafür, ich hab' deinen Beitrag im anderen Forum aber echt nicht mehr wiedergefunden... ich werde schon krass vergesslich.)
Etwas schlechter ist, dass die Forderung im ersten Zitat von dir meine Position leicht verzerrt ausdrückt - ich plädiere ja nicht dafür, die Universumsexpansion generell als Geschwindigkeitseffekt zu deuten, sondern dafür, sich dieser Deutung bewusst zu bleiben, wenn man sie mal brauchen kann. Was sehr oft der Fall ist. Kein Problem soweit.
Später habe ich aber deine ähnlichlautende Forderung an Marco Polo mir der an mich gerichteten verwechselt, und das war ganz schlecht. Die Position, kosmologische Rotverschiebung sei auf allen Skalen als Dopplereffekt zu deuten, ist nämlich fast entgegengesetzt zu meiner - ich will ja nur, dass man die für das Problem am besten passende Interpretation verwendet und keine verbietet. Das habe ich auch immer und immer wieder betont. Daher meine Frustration.

Entschuldigung, wenn ich dich da falsch verstanden habe.

Marco Polo
24.02.14, 00:24
Hallo Timm,

deinen folgenden Beitrag hatte ich ganz übersehen.

Angenommen, es gelingt eine Abstandsänderung zwischen 2 inertialen Objekten zu detektieren. Vielleicht nicht den exakten Abstand zu festen Zeitpunkten, aber zumindest 'nimmt zu' oder 'nimmt ab'. Welches Kriterium bräuchte man dann, um diesen Befund als Raumdehnung oder als Auseinander-Bewegung, oder als beides kombiniert zu unterscheiden?

Wenn ichs mir recht überlege, gibt es ein solches Kriterium nicht. Es wird daher mit der von mir vorgeschlagenen Messmethode schwierig bis unmöglich.

Bauhof
24.02.14, 10:03
Hallo Eugen,

zu diesem Thema habe ich im Nachbarforum unlängst geschrieben:

Rebhan [1] schreibt dazu: "Die Deutung der ART-Kosmologie, dass die Zunahme der Galaxienabstände ausschliesslich auf einer Expansion des Raumes beruht, ist auf das System der mitbewegten Koordinaten, für das sie abgeleitet wurde, beschränkt. Auch in der ART-Kosmologie kann man Koordinaten benutzen, in denen sich die Galaxien gegenüber dem Raum bewegen".

[1] Rebhan, Eckhard
Theoretische Physik: Relativitätstheorie und Kosmologie
Springer-Verlag Berlin Heidelberg 2012
ISBN=978-3-8274-2314-6

Grüsse, MP

Hallo Marc,

dieses Buch besitze ich auch, allerdings die 1. Auflage aus dem Jahr 1999. Ich finde das Zitat nicht in meiner Ausgabe. In welchem Kapitel und auf welcher Seite ist das Zitat in deiner Ausgabe zu finden?

M.f.G. Eugen Bauhof

Bauhof
24.02.14, 11:11
Hallo Marc

wie du weißt, habe ich bei Amazon einen Buchverkäufer-Account, deshalb kann in Büchern suchen, die dort aufblätterbar angeboten werden. Dein Zitat habe ich leider noch nicht gefunden, dafür aber etwa anders. Rebhan schreibt im Kapitel Kosmographie auf Seite 434 seines Buches (http://www.amazon.de/Theoretische-Physik-Relativit%C3%A4tstheorie-Kosmologie-German/dp/3827423147/ref=sr_1_1?s=books&ie=UTF8&qid=1393238062&sr=1-1)folgendes:

Man kann sich die für a(t)>0 in mitbewegten Koordinaten stattfindende Expansion des Universums so vorstellen, daß in dem die Galaxien trennenden Raumgebiet zwischen den bereits vorhandenen Raumelementen permanent und überall neue (infinitesimale) Raumelemente entstehen, die den Abstand zwischen den Galaxien vergrößern.

Wegen des kumulativen Effekts der Entstehung neuer Raumelemente ist diese Vergrößerung proportional zum Abstand der Galaxien, (Einschränkungen, die bei der Expansion bzw. Kontraktion zu beachten sind, werden im Kapitel 22 besprochen.) Mit dem Ballonmodell des Universums läßt sich das gut veranschaulichen.

Die Anzahl der Moleküle auf der – im Kleinen natürlich stark zerklüfteten – Oberfläche des Ballons wächst mit deren Flächeninhalt. Wird der Ballon aufgeblasen, so werden überall Moleküle von unterhalb der Oberfläche in diese mit einbezogen. Für ein in der Ballonoberfläche beheimatetes Flachlebewesen, das nichts von der Einbettung des Ballons in den R³ weiß, scheinen diese aus dem Nichts zu kommen.

Auch für uns als dreidimensionale Wesen scheint unsere "3-D-Raumvermehrung" bei der Universum-Expansion aus dem Nichts zu kommen

M.f.G. Eugen Bauhof

Marco Polo
24.02.14, 11:59
dieses Buch besitze ich auch, allerdings die 1. Auflage aus dem Jahr 1999. Ich finde das Zitat nicht in meiner Ausgabe. In welchem Kapitel und auf welcher Seite ist das Zitat in deiner Ausgabe zu finden?


Da muss ich dich leider bis FR vertrösten, Eugen. Bin erst dann wieder zu Hause. Hoffentlich finde ich das Zitat auf die Schnelle überhaupt wieder. Das Buch hat ja immerhin ca. 600 Seiten. :rolleyes:

Bauhof
24.02.14, 13:17
Da muss ich dich leider bis FR vertrösten, Eugen. Bin erst dann wieder zu Hause. Hoffentlich finde ich das Zitat auf die Schnelle überhaupt wieder. Das Buch hat ja immerhin ca. 600 Seiten. :rolleyes:

Hallo Marc,

du musst nicht mehr das Zitat suchen, ich habe es bei Amazon in der neuen Ausgabe nun gefunden. Es war leichter als gedacht, ich musste nur den Suchbegriff "Galaxienabstände" eingeben und schon wird die entsprechende Seite aufgeblättert. Das funktioniert leider nur dann, wenn bei Amazon einen Account hat. Hier nochmals dein Zitat auf Seite 433 des Buches von Rebhan (http://www.amazon.de/Theoretische-Physik-Relativit%C3%A4tstheorie-Kosmologie-German/dp/3827423147/ref=sr_1_1?s=books&ie=UTF8&qid=1393238062&sr=1-1)in einem größeren Zusammenhang:

Die Deutung der ART-Kosmologie, daß die Zunahme der Galaxienabstände ausschließlich auf einer Expansion des Raums beruht, ist auf das System der mitbewegten Koordinaten χ, δ und ϕ, für das sie abgeleitet wurde, beschränkt. Auch in der ART-Kosmologie kann man Koordinaten benutzen, in denen sich die Galaxien gegenüber dem Raum bewegen. Insbesondere muß es im Fall hinreichend niedriger Galaxiendichte Koordinaten geben, in denen die Galaxiengeschwindigkeiten in der Nähe des Koordinatenursprungs durch das Ergebnis (15.9) der Newton-Theorie beschrieben werden.

Die zugehörige Transformation besteht allerdings nicht einfach in einer Umkehrung der Transformation (15.15),...

Ich interpretiere das wie folgt:
Wenn man ein bestimmtes Koordinatensystem zur Beschreibung der Universum-Expansion benutzt, dann kann man die Galaxien als bewegt gegenüber dem Raum ansehen.

Das kann man sicherlich, aber die Standard-Interpretation ist bislang immer noch das Ballon-Modell, auf deren Oberfläche die Galaxien ruhen, so wie es auch Rebhan an andere Stelle beschreibt. Siehe dazu diesen Beitrag.

(http://www.quanten.de/forum/showpost.php5?p=74812&postcount=85)M.f.G. Eugen Bauhof

P.S.
Das Buch hat bereits in meiner alten Ausgabe nicht 600, sondern 1222 Seiten.

Bauhof
24.02.14, 13:45
Das schöne ist, dass Marco Polo mittlerweile mit Rebhan tatsächlich eine Quelle aufgetan hat, die meine Sichtweise ausdrückt.

Hallo Ich,

siehe dazu meinen Beitrag (http://www.quanten.de/forum/showpost.php5?p=74817&postcount=87).

– ich plädiere ja nicht dafür, die Universumsexpansion generell als Geschwindigkeitseffekt zu deuten, sondern dafür, sich dieser Deutung bewusst zu bleiben, wenn man sie mal brauchen kann. Was sehr oft der Fall ist. Kein Problem soweit.
[...]
Die Position, kosmologische Rotverschiebung sei auf allen Skalen als Dopplereffekt zu deuten, ist nämlich fast entgegengesetzt zu meiner – ich will ja nur, dass man die für das Problem am besten passende Interpretation verwendet und keine verbietet. Das habe ich auch immer und immer wieder betont.

Ja, ich denke, für kleine Abstände zwischen den Galaxien ist die Deutung der Universum-Expansion als Geschwindigkeitseffekt zulässig und einfacher. Auf großen Skalen aber nicht mehr.

M.f.G. Eugen Bauhof

JoAx
24.02.14, 14:54
Schauen wir mal, was [Landafshitz] dazu schreiben:

Chapter 14. Relativistic cosmology
§114 The red shift

The "red shift" phenomenon is essentially the Doppler effect of the galaxies' "running away" from each other.


[Landafshitz]: Landau L.D., Lifshitz E.M. Course of theoretical physics Vol. 2. The classical theory of fields 1994

JoAx
24.02.14, 15:22
Wie kommst du in Bezug zur Expansion darauf oder woher hast du das? (Irgendwo aufgeschnappt?)


Wie komme ich worauf?

Timm
24.02.14, 15:22
Da muss ich dich leider bis FR vertrösten, Eugen. Bin erst dann wieder zu Hause. Hoffentlich finde ich das Zitat auf die Schnelle überhaupt wieder. Das Buch hat ja immerhin ca. 600 Seiten. :rolleyes:
http://books.google.de/books?id=-iz1Rhv8wLMC&pg=PA433&lpg=PA433&dq=Auch+in+der+ART-Kosmologie+kann+man+Koordinaten+benutzen,+in+denen +sich+die+Galaxien+gegen%C3%BCber+dem+Raum+bewegen&source=bl&ots=p2wqJw42Ya&sig=mmeMMJCTWzJg6lbOS1yVQLGSFIM&hl=de&sa=X&ei=S1sLU8jXIMmMtQaUxYGwBQ&redir_esc=y#v=onepage&q=Auch%20in%20der%20ART-Kosmologie%20kann%20man%20Koordinaten%20benutzen%2 C%20in%20denen%20sich%20die%20Galaxien%20gegen%C3% BCber%20dem%20Raum%20bewegen&f=false

Deshalb ist es doch gut, daß es das Internet gibt, Marc. :)

Hier nochmal das Zitat:

Die Deutung der ART-Kosmologie, dass die Zunahme der Galaxienabstände ausschliesslich auf einer Expansion des Raumes beruht, ist auf das System der mitbewegten Koordinaten ., . und ., für das sie abgeleitet wurde, beschränkt. Auch in der ART-Kosmologie kann man Koordinaten benutzen, in denen sich die Galaxien gegenüber dem Raum bewegen.

Ich finde mich da ebenfalls wieder. Wie sollte man diese Feststellung auch sachlich widerlegen.

JoAx
24.02.14, 15:30
Wobei man bei "dem Raum" nie ausser Acht lassen darf, dass es sich dabei um "Raum" der speziell gewählten Koordinaten handelt, und nicht um eine "eigenständige Entität".

Marco Polo
24.02.14, 17:26
Deshalb ist es doch gut, das es das Internet gibt, Marc. :)

Danke Timm. Das wusste ich ja noch gar nicht. Wieso hat mir das nie jemand gesagt? :D

Günter Panazée
24.02.14, 22:00
Genau diese Art von Einsicht ist es die man von diesem Personenkreis erwarten kann:

Hier nochmal das Zitat:

Die Deutung der ART-Kosmologie, dass die Zunahme der Galaxienabstände ausschliesslich auf einer Expansion des Raumes beruht, ist auf das System der mitbewegten Koordinaten ., . und ., für das sie abgeleitet wurde, beschränkt. Auch in der ART-Kosmologie kann man Koordinaten benutzen, in denen sich die Galaxien gegenüber dem Raum bewegen.

Ich finde mich da ebenfalls wieder. Wie sollte man diese Feststellung auch sachlich widerlegen.

Man findet sich in diesem Zitat wieder - Im anderen Rebhan-Zitat dann wohl anscheinend nicht.

Schauen wir mal, was [Landafshitz] dazu schreiben:

Chapter 14. Relativistic cosmology
§114 The red shift

The "red shift" phenomenon is essentially the Doppler effect of the galaxies' "running away" from each other.

[Landafshitz]: Landau L.D., Lifshitz E.M. Course of theoretical physics Vol. 2. The classical theory of fields 1994

Man kann wunderbar sinnentstellend zitieren wenn man den Kontext ausblendet und sich auf das "passende Ergebnis" konzentriert - Aus gleicher Quelle:
We shall see that a basic feature of this model is its nonstationarity. There is no doubt that this property gives a correct explanation of such a fundamental for the entire cosmology phenomnon as the "red shift" ....

Wie ich schon sagte: Ich diskutiere mit solchen Leuten nicht mehr, das hat gar keinen Sinn.

Wie komme ich worauf?

Vergiss es: Mich interessiert es nicht mehr wie du auf irgendetwas kommst.

Frage: Kann ich bzw. ein Admin mich und meine Beiträge hier wieder löschen?
Dann bitte ich darum.

Gonzee

JoAx
24.02.14, 22:32
Man kann wunderbar sinnentstellend zitieren wenn man den Kontext ausblendet und sich auf das "passende Ergebnis" konzentriert


Das ist eine Lüge. Ich habe nichts ausgelassen und den Kontext durch die Angabe der Quelle auch hergestellt. Du hast dagegen unvollständig zitiert:

We shall see that a basic feature of this model is its nonstationarity. There is no doubt that this property ("the expanding universe") gives a correct explanation of such a fundamental for the entire cosmology phenomenon as the "red shift", which is fundamental for the cosmological problem.

Es ist das Universum, das expandiert, und nicht Raum.


Frage: Kann ich bzw. ein Admin mich und meine Beiträge hier wieder löschen?
Dann bitte ich darum.


Ja, dass ist für einen gewissen (lern-resistenten) Personenkreis typisch.

Marco Polo
25.02.14, 00:53
Wie ich schon sagte: Ich diskutiere mit solchen Leuten nicht mehr, das hat gar keinen Sinn.

Was soll denn das jetzt bitte heißen? Solche Leute. Wer sind denn solche Leute? Wir etwa?

Dabei ist doch über die Landesgrenzen hinaus bekannt, dass wir hier ein Privatzirkel sind, der abstrusen Spezialtheorien huldigt. :rolleyes:

Marco Polo
25.02.14, 00:57
Frage: Kann ich bzw. ein Admin mich und meine Beiträge hier wieder löschen? Dann bitte ich darum.

Wir könnten dich einfach sperren wenn dies dein Wunsch ist... :rolleyes:

Ach. Ich hab die Sperre jetzt einfach mal vollzogen...

soon
25.02.14, 07:38
Hi,
Wobei man bei "dem Raum" nie ausser Acht lassen darf, dass es sich dabei um "Raum" der speziell gewählten Koordinaten handelt, und nicht um eine "eigenständige Entität".

Das ist nicht scharf genug.

Solange man sich die Koordinaten beliebig ausdenken kann, bleibt es Mathematik und ist keine Physik.

In der Natur herrscht Quantelung. Deshalb muß an den gewählten Koordinaten ganz konkret und tatsächlich etwas vorhanden sein.
Es ist wichtig den Aspekt der Quantelung mit einzubeziehen.

Zur Verdeutlichung: Dies steht in einer gewissen Analogie z.B. zum Photoeffekt, bei der auch nicht jedes Photon, mit beliebig ausgedachten Eigenschaften, mit jedem Atom wechselwirken kann.

LG soon

JoAx
25.02.14, 08:36
Solange man sich die Koordinaten beliebig ausdenken kann, bleibt es Mathematik und ist keine Physik.


Das ist nicht korrekt, soon. Es bedeutet, dass die Physik nicht von Koordinatenwahl abhängt. Man muss Sachen suchen, die koordinatenunabhängig, invariant sind.


In der Natur herrscht Quantelung. Deshalb muß an den gewählten Koordinaten ganz konkret und tatsächlich etwas vorhanden sein.


Das würde aber dem Prinzip, dass die Physik nicht von der Koordinatenwahl abhängen darf, widersprächen.

Ich
25.02.14, 08:57
Ja, ich denke, für kleine Abstände zwischen den Galaxien ist die Deutung der Universum-Expansion als Geschwindigkeitseffekt zulässig und einfacher. Auf großen Skalen aber nicht mehr.
Eine Geschwindigkeitsangabe ist sicher auch bei großen Skalen zulässig. Ich kann mir aber nur wenige Fälle vorstellen, in denen das etwas bringt, von daher wird man davon nicht oft Gebrauch machen. Spätestens im geschlossenen Universum wäre es auch nicht für die globale Beschreibung geeignet, da muss man eigentlich zwangsläufig das Bild vom Luftballon benutzen (aber nur mit darauf schwimmenden Galaxien, nicht festgeklebten!).

Es ist ja auch zulässig, auf kleinen Skalen von Raumdehnung zu reden. Das ändert nichts daran, dass eine solche Beschreibung im besten Falle unbrauchbar, im schlechtesten total irreführend ist. Jede Beschreibung hat ihre Vor- und Nachteile.

soon
25.02.14, 09:19
Es bedeutet, dass die Physik nicht von Koordinatenwahl abhängt.

Das sehe ich anders. Du kannst die Koordinaten frei wählen. Aber nur innerhalb der Menge der Koordinaten, die physikalisch vorhanden sind.

Erdmittelpunkt ist eine Koordinate, Mondmittelpunkt ist eine Koordinate, der "Raumpunkt" auf halben Weg dazwischen ist keine Koordinate, denn er läßt sich genau genommen gar nicht eindeutig bestimmen.

LG soon

JoAx
25.02.14, 09:31
Das sehe ich anders. Du kannst die Koordinaten frei wählen. Aber nur innerhalb der Menge der Koordinaten, die physikalisch vorhanden sind.

Erdmittelpunkt ist eine Koordinate, Mondmittelpunkt ist eine Koordinate, der "Raumpunkt" auf halben Weg dazwischen ist keine Koordinate, denn er läßt genau genommen gar nicht eindeutig bestimmen.

LG soon

Dann würdest du so was:

Kruskal-Szekeres-Koordinaten (http://de.wikipedia.org/wiki/Kruskal-Szekeres-Koordinaten)

ablehnen?

soon
25.02.14, 10:31
Dann würdest du so was:

Kruskal-Szekeres-Koordinaten (http://de.wikipedia.org/wiki/Kruskal-Szekeres-Koordinaten)

ablehnen?

Ablehnen?, nein. Eine Gleichsetzung von Mathematik und Physik lehne ich ab, das ist richtig.

Und die Frage nach einer "Ausdehnung des Raums" ist für mich eher Mathematik, solange keine genaue Formulierung eines ganz konkreten physikalische Problems hinzukommt.

Das, nämlich, führt nur zu überflüssigen Streitereien.

LG soon

Timm
25.02.14, 16:19
Rebhan schreibt im Kapitel Kosmographie auf Seite 434 seines Buches (http://www.amazon.de/Theoretische-Physik-Relativit%C3%A4tstheorie-Kosmologie-German/dp/3827423147/ref=sr_1_1?s=books&ie=UTF8&qid=1393238062&sr=1-1)folgendes:
Man kann sich die für a(t)>0 in mitbewegten Koordinaten stattfindende Expansion des Universums so vorstellen, daß in dem die Galaxien trennenden Raumgebiet zwischen den bereits vorhandenen Raumelementen permanent und überall neue (infinitesimale) Raumelemente entstehen, die den Abstand zwischen den Galaxien vergrößern.

Rebhan äußert sich auch hier sehr sachlich, Eugen. Er schreibt nicht, daß es so ist, sondern man sich die Expansion bei Verwendung mitbewegter Koordinaten so vorstellen kann.
Wie man sich etwa auch den Durchgang einer Gravitationswelle als Stauchen/Dehnen von Raum vorstellen kann. Auch hier wird allerdings häufig auf die Koordinatenabhängigkeit solcher Vorstellungen hingewiesen.
Ein krasses Beispiel für eine der menschlichen Intuition zuhilfe kommende Vorstellung bringt der Kosmologe Edward R. Harrison in seinem Buch "Kosmologie", 1984. Danach fällt der Raum mit Lichtgeschwindigkeit über den Ereignishorizont eines schwarzen Loches. Dann ist klar, warum das Photon bei r = 2M stationär ist, obwohl es sich lokal mit c bewegt. :D

Gruß, Timm

G. Pannazée
26.02.14, 09:14
Du hast dagegen unvollständig zitiert:

Ich habe es nochmals geprüft: In der third revised english edition und in dieser steht es exakt so wie zitiert drin.

Aber auch unter Zugrundelegung Deiner "Version":

Was sollen sich dadurch konkret an der inhaltlichen Aussage des Zitats ändern, Johann?
Etwa dass das Bewegungsmodell gültig ist?
Sinngemäß steht dort wie dort "Es ist zweifellos die Eigenschaft mitbewegter Koordinaten die die korrekte Erklärung liefert"
-> Wen willst Du hier auf den Arm nehmen?

Ein kinetisches Bewegungsmodell in statischen Koordinaten ist weder mit der Urknalltheorie verträglich noch steht es mit den CMB-Beobachtungen im Einklang -
Nicht umsonst ist die Raumzeit-Expansion Bestandteil des ΛCDM-Modells und nicht das Bewegungsmodell.

Entweder legst Du jetzt sofort Alternativ-Theorien /-Erklärungen für diese beiden Aspekte auf den Tisch oder Du beschäftigst dich erst einmal mit den Grundlagen der ART evor Du dich an der Aufstellung alternativer Privattheorien versuchst.

Wer sind denn solche Leute? Wir etwa?

Ich kann nicht nachvollziehen warum du dich angesprochen fühlst:
In einem vorangegangenen Beitrag hatte ich auf Nachfrage von Eugen klargestellt dass man nicht alle User über einen Kamm scheren kann:
Mit "Leute" meinte ich ausschließlich Timm und Johann (Das was <Ich> anschließend noch geschrieben hatte fand ich z.B. in Ordnung)

Dabei ist doch über die Landesgrenzen hinaus bekannt, dass wir hier ein Privatzirkel sind, der abstrusen Spezialtheorien huldigt.

Auf Basis Deines hier gezeigten Verhaltens betrachte ich diese Feststellung mit weitaus weniger Ironie als Du Dir das vermutlich beim Verfassen gedacht hast.

Rebhan äußert sich auch hier sehr sachlich, Eugen. Er schreibt nicht, daß es so ist, sondern man sich die Expansion bei Verwendung mitbewegter Koordinaten so vorstellen kann.
Wie man sich etwa auch den Durchgang einer Gravitationswelle als Stauchen/Dehnen von Raum vorstellen kann. Auch hier wird allerdings häufig auf die Koordinatenabhängigkeit solcher Vorstellungen hingewiesen.

Gravitationswellen interpretiert man als Raumdehnung/stauchung.
Die Raumexpansion interpretiert man als Wachstum - Darauf hatte ich dich schon hingewiesen.
Bereits daran, dass Du die Begrifflichkeiten ohne Sinn und Verstand synonym verwendest und wild durcheinanderschmeisst sieht man, dass Du deren Hintergründe nicht verstanden hast.
Ich empfehle Dir erst einmal die ART zu lernen bevor Du dich an der Aufstellung alternativer Privattheorien versuchst.

Ein krasses Beispiel für eine der menschlichen Intuition zuhilfe kommende Vorstellung bringt der Kosmologe Edward R. Harrison in seinem Buch "Kosmologie", 1984. Danach fällt der Raum mit Lichtgeschwindigkeit über den Ereignishorizont eines schwarzen Loches. Dann ist klar, warum das Photon bei r = 2M stationär ist, obwohl es sich lokal mit c bewegt. :D

Für mich ist das Beispiel für einen, der die allgemeine Kovarianz der ART anwendet, gegenüber einem, der die allgemeine Kovarianz offenbar nicht verstanden hat.
Ein Physiker denkt sich bei dem von Dir geschilderten Sachverhalt nämlich maximal (wenn überhaupt) folgendes: "Harrison verwendet also mitbewegte Koordinaten - O.K."
(Und nicht "ein krasses Beispiel ...")

-> Du hast keine Ahnung aber davon verdammt viel.
Lerne erst einmal die ART - Sonst wird das auch zukünftig immer nur mit einem Griff ins Klo enden.

So - jetzt darfst Du mich wieder sperren, Marco:
Sowas bereitet Dir doch Freude.
Wie kann ich auch nur so unverschämt sein Deinen Freunden ans Bein zu pinkeln?

Mein Wunsch lautete im Übrigen mich und meine Beiträge hier zu löschen und nicht nur zu sperren -
Hatte ich mich da etwa unklar ausgedrückt?

Gonzee

Bauhof
26.02.14, 10:07
So - jetzt darfst Du mich wieder sperren, Marco:
Sowas bereitet Dir doch Freude. Wie kann ich auch nur so unverschämt sein Deinen Freunden ans Bein zu pinkeln?

Mein Wunsch lautete im Übrigen mich und meine Beiträge hier zu löschen und nicht nur zu sperren – Hatte ich mich da etwa unklar ausgedrückt?

Hallo G. Pannazée,

zu deiner Information:

1. Beiträge werden nur dann gelöscht, wenn sie gegen die Forenregeln verstoßen – gegen die geschriebenen und gegen die ungeschriebenen Forenregeln. Deshalb wurde deinem Wunsch nicht entsprochen.

2. Gesperrt hat dich Marc deswegen, weil du dauernd persönliche Angriffe in deinen Einträgen einbaust. Lass das doch bitte in Zukunft, dann könntest du vielleicht auch bleiben.

3. Ich habe dir noch eine Chance gegeben, dein Verhalten zu ändern. Eigentlich hätte ich deinen neuen Account sofort sperren müssen, weil Doppelaccounts nicht erlaubt sind. Ich habe noch das Recht dazu, obwohl ich kein Moderator mehr bin. Bitte entschuldige dich bei Timm und Johann, vielleicht ist dann noch was zu retten.

M.f.G. Eugen Bauhof

Marco Polo
26.02.14, 11:51
So - jetzt darfst Du mich wieder sperren, Marco:
Sowas bereitet Dir doch Freude.

Was für ein Unsinn.

Du hattest doch um Löschung deiner Beiträge und deines Accounts hier gebeten.

Löschen kannst du deine Beiträge selber.

Den Account kann man nicht löschen. Daher die Sperre.

Wenn jemand um Löschung seiner Beiträge und seines Accounts bittet, dann wird er eben gesperrt. Insbesondere dann, wenn dieser Jemand hier so herumpöbelt. Wir pflegen hier nämlich einen freundlichen Umgangston untereinander.

Ansonsten sehe ich es so wie Eugen:

3. Ich habe dir noch eine Chance gegeben, dein Verhalten zu ändern. Eigentlich hätte ich deinen neuen Account sofort sperren müssen, weil Doppelaccounts nicht erlaubt sind. Ich habe noch das Recht dazu, obwohl ich kein Moderator mehr bin. Bitte entschuldige dich bei Timm und Johann, vielleicht ist dann noch was zu retten.

JoAx
26.02.14, 13:05
In der third revised english edition und in dieser steht es exakt so wie zitiert drin.


Das kann natürlich sein. Ich habe aus der 4. Edition zitiert.


Was sollen sich dadurch konkret an der inhaltlichen Aussage des Zitats ändern, Johann?
Etwa dass das Bewegungsmodell gültig ist?


Jein.


Sinngemäß steht dort wie dort "Es ist zweifellos die Eigenschaft mitbewegter Koordinaten die die korrekte Erklärung liefert"


Landafshic sprechen nicht von mitbewegten Koordinaten, sondern vom nichtstationären Modell des Universums bei homogener und isotroper Materieverteilung über den Raum. Als ein "Gegenmodell" dazu kann man das Modell eines stationären Universums betrachten, das Einstein seinerzeit zunächst entwickelte. "Koordinaten" haben damit nichts zu tun. In dieser Grafik:

http://images.astronet.ru/pubd/2003/11/10/0001194831/map1a.gif

sieht man bsw. deutlich, dass die Fundamentalbeobachter auseinander laufen. Es ist keine alternative Erklärung, kein alternatives Modell. Es ist das selbe, wie das hier:

http://images.astronet.ru/pubd/2003/11/10/0001194831/map1b.gif

Die "mitbewegten Koordinaten" (comoving distance) wurden lediglich durch die "Eigenentfernung" (proper distance) ersetzt.

(Ich kann nichts dafür, dass die sprachliche Charakteresierung des Verhaltens der Weltlinien -- sie laufen auseinander -- exakt ist, sich aber auch mit dem "gemeinen Bild über gewöhnliche Bewegung im Raum" exakt deckt. :( Da muss man halt durch. Sich u.U. mit der Struktur der "Galilei-Raumzeit" beschäftigen.)


-> Wen willst Du hier auf den Arm nehmen?


Niemanden. Ich kann nur wiederholen, dass deine Einstellung zu dem, was hier gerade "abläuft", einfach falsch ist. Das einzige, was von dir bislang kam, was man als konstruktiv bezeichnen könnte ist das hier:


... Du beschäftigst dich erst einmal mit den Grundlagen der ART ...


gewesen. Ich hoffe, dass ich damit zu dir nun durchgedrungen bin.


Nicht umsonst ist die Raumzeit-Expansion Bestandteil des ΛCDM-Modells und nicht das Bewegungsmodell.


Meinst du nun wirklich die Raumzeit-Expansion, oder Raum-Expansion?


Gruß, Johann

Bauhof
27.02.14, 12:57
Interessant ist aber finde ich die Frage, ob eine relative Geschwindigkeit zwischen fundamentalen Beobachtern unabhängig von der Wahl der Koordinaten nur ein Teil der kosmologischen Rotverschiebung ist.

Hallo Timm,

wie kommst du auf diese Frage?

Was Sinn macht, denn in der SRT ist die Rotverschiebung global ein Dopplereffekt, in der ART nur lokal. Demnach sollte mit Letzterer ein von der Raumzeitkrümmung abhängiger Anteil der Rotverschiebung hinzu kommen.

Was meinst du hier mit Raumzeitkrümmung? Die lokale Raumzeitkrümmung der einzelnen Galaxien? Oder irgendeine globale Krümmung, wie z.B. die Weyl-Krümmung (http://de.wikipedia.org/wiki/Weyl-Kr%C3%BCmmung)?

M.f.G. Eugen Bauhof

G. Pannazée
28.02.14, 08:11
Du hattest doch um Löschung deiner Beiträge und deines Accounts hier gebeten.
Löschen kannst du deine Beiträge selber.
Den Account kann man nicht löschen. Daher die Sperre.

Verstehe:
Auf meine Bitte hin mich und meine Beiträge zu löschen,
sperrst Du mich um mir hinterher mitzuteilen dass
ich meine Beiträge selbst hätte löschen können -
Was allerdings nur geht wenn man noch nicht gesperrt ist.

Absolut nachvollziehbare Logik.

Was für ein Unsinn.

Gleichfalls.

Eigentlich hätte ich deinen neuen Account sofort sperren müssen, weil Doppelaccounts nicht erlaubt sind.
*Ich habe noch das Recht dazu, obwohl ich kein Moderator mehr bin.*

Wie bitte? Heißt das Du hast das Recht andere User zu sperren obwohl Du nur ein "normaler" User bist?
Verstehe.

Insbesondere dann, wenn dieser Jemand hier so herumpöbelt. Wir pflegen hier nämlich einen freundlichen Umgangston untereinander.

Wen meinst Du mit "untereinander"?

Machen wir es kurz:
Ich werde diese eingeschworene Bruderschaft nicht länger belästigen:
Ihr kommt sicher besser ohne mich klar - Genauso wie andersherum.

Gonzee

P.S.:
Meinst du nun wirklich die Raumzeit-Expansion, oder Raum-Expansion?

"Wachstum" ist üblicherweise eine Funktion F(t) - Was Deine Frage erst einmal beantworten sollte (und fraglos gleichzeitig interessante andere aufwirft).

Marco Polo
28.02.14, 08:48
Verstehe:
Auf meine Bitte hin mich und meine Beiträge zu löschen,
sperrst Du mich um mir hinterher mitzuteilen dass
ich meine Beiträge selbst hätte löschen können -
Was allerdings nur geht wenn man noch nicht gesperrt ist.

Absolut nachvollziehbare Logik.

Nun. Eigentlich bin ich davon ausgegangen, dass selbst dem Einfältigsten klar sein sollte, dass sich meine Info bezüglich des Löschens deiner Beiträge nur auf deinen neuen noch nicht gesperrten Account beziehen konnte.
Sorry, wenn ich dich da intellektuell überfordert haben sollte.

Wie bitte? Heißt das Du hast das Recht andere User zu sperren obwohl Du nur ein "normaler" User bist?
Verstehe.

Eugen Bauhof hat Sonderrechte. Im Prinzip ist er Moderator.

Machen wir es kurz:
Ich werde diese eingeschworene Bruderschaft nicht länger belästigen:
Ihr kommt sicher besser ohne mich klar - Genauso wie andersherum.

Das sehe ich genauso.

G. Pannazée
28.02.14, 09:10
Nun. Eigentlich bin ich davon ausgegangen, dass selbst dem Einfältigsten klar sein sollte, dass sich meine Info bezüglich des Löschens deiner Beiträge nur auf deinen neuen noch nicht gesperrten Account beziehen konnte.
Sorry, wenn ich dich da intellektuell überfordert haben sollte.

Damit bin ich in der Tat intellektuell überfordert:
Meine Bitte formulierte ich schließlich unter meinem alten Account (und sie bezog sich auf meine alten Beiträge), der gesperrt wurde bevor ich dem Hinweis, die Beiträge selbst zu löschen, nachkommen konnte.

Eugen Bauhof hat Sonderrechte. Im Prinzip ist er Moderator.

Im Prinzip schon aber gleichzeitig auch nicht - Das habe ich verstanden, Danke.

EOD.

Gonzee

Bauhof
28.02.14, 10:01
Zitat von Bauhof:
Eigentlich hätte ich deinen neuen Account sofort sperren müssen, weil Doppelaccounts nicht erlaubt sind.
*Ich habe noch das Recht dazu, obwohl ich kein Moderator mehr bin.*

Wie bitte? Heißt das Du hast das Recht andere User zu sperren obwohl Du nur ein "normaler" User bist?
Verstehe.

Hallo G. Pannazée,

genau das ist leider dein Problem, die urteilst, ohne dich vorher zu informieren, was Sache ist.

Als du nach den Forenmitarbeitern gefragt hast, hatte ich dir angeboten, mittels einer PN dich darüber zu informieren. Das hast du abgelehnt.

Nicht wir müssen uns ändern, sondern du. Deshalb denke ich jetzt, dass da nichts mehr zu retten ist.

M.f.G. Eugen Bauhof

Timm
28.02.14, 10:10
Hallo Eugen,

Zitat von Timm

Interessant ist aber finde ich die Frage, ob eine relative Geschwindigkeit zwischen fundamentalen Beobachtern unabhängig von der Wahl der Koordinaten nur ein Teil der kosmologischen Rotverschiebung ist.
wie kommst du auf diese Frage?
Durch die Publikationen, die ich an anderer Stelle aufgelistet hatte. Insbesondere interessant ist A diatribe on expanding space ( http://arxiv.org/pdf/0809.4573v1.pdf). Gleichung (16) zeigt die Rotverschiebung als Produkt eines Doppler- und eines gravitativen Terms. Im idealisierten Fall (homogene Massendichte) folgt daraus, daß z sich einer "reinen" Dopplerverschiebung annähert, wenn die Differenz der Zeitpunkte von Emission und Absorption gegen Null geht. Andere Autoren verfolgen alternative Ansätze, wie die Parallelverschiebung der vierer-Geschwindigkeit auf der Null-Geodäte zwischen Emission und Absorpton.

Peacock verweist eingangs auf de Sitter. Die von diesem abgeleitete Kosmologie ist statisch, Gleichung (2), in RW Koordinaten (1) expandiert sie. Auch wenn die Energiedichte hier nur lambda enthält, deutlicher läßt sich nicht zeigen, daß es bei dieser Diskussion nicht um richtig oder falsch geht und schon gar nicht um eine neue Theorie, wie der user G.P. in Unkenntnis behauptet.

[
Was meinst du hier mit Raumzeitkrümmung? Die lokale Raumzeitkrümmung der einzelnen Galaxien? Oder irgendeine globale Krümmung, wie z.B. die Weyl-Krümmung (http://de.wikipedia.org/wiki/Weyl-Kr%C3%BCmmung)?

Nicht die Galaxien, die Bemerkung bezieht sich auf das FRW Modell mit homogener Massendichte, Ricci Tensor <> 0.

Gruß,Timm

JoAx
28.02.14, 10:50
"Wachstum" ist üblicherweise eine Funktion F(t) - Was Deine Frage erst einmal beantworten sollte (und fraglos gleichzeitig interessante andere aufwirft).

Genial!

Die Koordinate eines materiellen Objektes kann auch eine Funktion der Zeit sein und nennt sich Geschwindigkeit -- x(t)=v. Meine Frage wurde damit also nicht beantwortet.

Hawkwind
28.02.14, 13:04
Genial!

Die Koordinate eines materiellen Objektes kann auch eine Funktion der Zeit sein und nennt sich Geschwindigkeit -- x(t)=v. Meine Frage wurde damit also nicht beantwortet.

Ich hätte gedacht, x(t) nennt sich "Ort" und nicht "Geschwindigkeit".

Marco Polo
28.02.14, 22:06
Es muss natürlich heissen: x(t)=vt

Das ist das Zeit-Orts-Gesetz der gleichförmigen Bewegung.

Marco Polo
28.02.14, 22:50
Damit bin ich in der Tat intellektuell überfordert:

Wäre ich gemein würde ich mutmaßen, dass sich dieser Umstand wie ein roter Faden durch deine bisherige "wissenschaftliche Karriere" zieht. Bin ich aber nicht. :)

Meine Bitte formulierte ich schließlich unter meinem alten Account (und sie bezog sich auf meine alten Beiträge), der gesperrt wurde bevor ich dem Hinweis, die Beiträge selbst zu löschen, nachkommen konnte.Das stimmt. Stimmen tut aber auch, dass sich mein Löschhinweis auf deine möglichen Folgebeiträge unter neuem Account bezog. Auf was auch sonst?

Du hattest um Löschung deiner Beiträge und Löschung deines Accounts gebeten. Bevor du aber selbst darauf gekommen wärst, wie man seine Beiträge löscht, habe ich die Sperre vollzogen um deine bisherigen Beiträge zu sichern. Sieh es einfach als Wertschätzung und nicht als Bestrafung.

Es ist nämlich so, dass alle Beiträge hier Eigentum des Forums sind. Das Recht hier zu schreiben bzw. seine Beiträge zu löschen besteht nur solange, wie es die Forenleitung gewährt.

Klar soweit?

Im Prinzip schon aber gleichzeitig auch nicht - Das habe ich verstanden, Danke.Na bitte. Es geht doch. ;)

p.s. und nochmal: entgegen deiner Behauptung, dass mir die Sperrung deines Accounts Freude bereitet ist es vielmehr so, das ich dies bedaure. Viel lieber wäre mir, wenn du die Diskussion (auf sachlicher Ebene) fortführen würdest.

JoAx
01.03.14, 14:17
Ihr habt Recht, Jungs!

Ich formuliere es anders. Mag schon sein, dass Wachstum durch eine zeitabhängige Funktion F(t) beschrieben wird. Daraus folgt aber noch lange nicht, dass jede zeitabhängige Funktion auch Wachstum beschreiben muss.

Die Frage bleibt offen.

Grüße

G. Pannazée
02.03.14, 07:11
Daraus folgt aber noch lange nicht, dass jede zeitabhängige Funktion auch Wachstum beschreiben muss.

Herausragende Erkenntnis.

Die Frage bleibt offen.
Eine korrekt gestellte Frage beinhaltet zu 2/3 die Antwort.

Meinst du nun wirklich die Raumzeit-Expansion, oder Raum-Expansion?"Wachstum" ist üblicherweise eine Funktion F(t) - Was Deine Frage erst einmal beantworten sollte (und fraglos gleichzeitig interessante andere aufwirft).


Nach kurzer Bemühung der Suchfunktion:

Bevor man eigene Theorien aufstellt, muss man sich mit bestehenden vertraut machen, und das nicht durch populärwissenschaftliche Artikeln. Du magst es nicht glauben, aber man sieht, dass du Null Ahnung hast. Die einzige konstruktive Kritik, die man dir an dieser Stelle geben kann, ist - schmeiss alles, was du dir selbst ausgedacht hast in den Heizofen, und fange an zu lernen.

http://en.wikipedia.org/wiki/Metric_expansion_of_space

Zu der von Dir gestellten Frage gibt insbesondere das Unterkapitel Metric tensor eine klare Antwort.

Für fachlich interessierten stillen Mitleser die wesentlichen Kernpunkte der metric expansion aus wikipedia nochmals grundsätzlich zitiert:
1. "The metric expansion of space is the increase of the distance between two distant parts of the universe with time.
It is an intrinsic expansion whereby the scale of space itself changes."
2. "Metric expansion is a key feature of Big Bang cosmology."


Damit bin ich in der Tat intellektuell überfordert:
Wäre ich gemein würde ich mutmaßen, dass sich dieser Umstand wie ein roter Faden durch deine bisherige "wissenschaftliche Karriere" zieht. Bin ich aber nicht.:)

Insbesondere dann, wenn dieser Jemand hier so herumpöbelt. Wir pflegen hier nämlich einen freundlichen Umgangston untereinander.
Wäre ich gemein würde ich auch etwas mutmaßen. Bin ich aber nicht.


Nicht wir müssen uns ändern, sondern du.

Das hatte ich Dir in einer PN geschrieben - Mit der Erlaubnis diese an deine Kollegen weiterzuleiten.
Wie du darauf kommst den Inhalt einer PN in einem öffentlich zugänglichen Thread kund zu tun hat weiss ich allerdings nicht.


Eugen Bauhof hat Sonderrechte. Im Prinzip ist er Moderator.

Laut dem Ergebnis der letzten Moderatorenwahl wurden vier Moderatoren gewählt:
Marco Polo
Ich
amc
Jogi


Dieses Forum hat allen Charme den der Gemeinschaftsraum eines Seniorenheimes bietet:
Hier fühlt sich jeder Gast auf Anhieb willkommen, pudelwohl und in die Gemeinschaft integriert.
Letztere zeichnet sich insbesondere durch ihre mit ihrer Eingeschworenheit einhergehende uneingeschränkte Neutralität und Objektivität gegenüber Dritten aus.


Prediger, die sich nicht an ihre eigenen Worte halten, sind Scharlatane.

Habe ich mich jetzt klar genug ausgedrückt und genügt das jetzt für (m)eine (weitere) Sperre? Danke.

Gonzee

P.S.:

Die "mitbewegten Koordinaten" (comoving distance) wurden lediglich durch die "Eigenentfernung" (proper distance) ersetzt.

(Ich kann nichts dafür, dass die sprachliche Charakteresierung des Verhaltens der Weltlinien -- sie laufen auseinander -- exakt ist, sich aber auch mit dem "gemeinen Bild über gewöhnliche Bewegung im Raum" exakt deckt.

Gleichzeitig unstrittig ist die allgemeine Kovarianz der GR

Und wer es bis jetzt noch nicht verstanden hat:
Das Standardmodell bricht mit der allgemeinen Kovarianz der GR indem mit der Urknalltheorie eine Interpretation bezorzugt wird.
Es spricht allerdings grundsätzlich nichts dagegen in Privattheorien von anderen Annahmen auszugehen.

(Für fachlich weitergehend interessierte Leser zum therma "cosmological distances" im Übrigen: http://en.wikipedia.org/wiki/Comoving_distance)

Ich
03.03.14, 09:54
GP,

wenn du dich hier nicht wohlfühlst, dann schreib einfach nicht. Du bist freiwillig hier, niemand hat dich eingeladen oder gar aufgefordert, hier mitzumachen.
Ständig hier zu schreiben, dass du hier nicht mehr schreiben wirst und gesperrt werden willst ist nicht altersgerecht. Allgemeine Kovarianz und ihre Anwendung (wie im dir bereits genannten Paper (http://arxiv.org/pdf/0809.4573.pdf)von Peacock) als Privattheorie zu titulieren und deswegen Leute blöd anzureden ist ebenfalls nicht akzeptabel.

Ich halte dir zugute, dass du vielleicht einfach nur missverstanden hast, was hier diskutiert wird, und geglaubt hast, in gerechtem Zorn hier so auftreten zu müssen.

Du kannst mit deinem nächsten Beitrag klarmachen, dass du sachlich mitdiskutieren willst und dich vorschneller Urteile und persönlicher Anfeindungen enthältst. Wenn du das nicht vorhast, bleib einfach weg. Dafür ist nicht erforderlich, ein nächstes Mal deine eigene Sperrung zu fordern oder ein nächstes Mal zu schreiben, dass du hier nicht schreiben wirst.

-Ich-

G. Pannazée
03.03.14, 12:07
Allgemeine Kovarianz und ihre Anwendung (wie im dir bereits genannten Paper (http://arxiv.org/pdf/0809.4573.pdf)von Peacock) als Privattheorie zu titulieren

Sinnentnehmendes Lesen ist offensichtlich nicht Deine Stärke: Siehe http://www.quanten.de/forum/showpost.php5?p=74789&postcount=69

-> Ich bin raus

Gonzee

JoAx
03.03.14, 12:25
Sinnentnehmendes Lesen ist offensichtlich nicht Deine Stärke: Siehe http://www.quanten.de/forum/showpost.php5?p=74789&postcount=69

-> Ich bin raus


Leider war das zu erwarten.
Kann mir dann vlt. irgend jemand anderes eine Quelle nennen, die das hier:


Das Standardmodell bricht mit der allgemeinen Kovarianz der GR indem mit der Urknalltheorie eine Interpretation bezorzugt wird.


belegen würde?

Ich
03.03.14, 21:27
Kann mir dann vlt. irgend jemand anderes eine Quelle nennen, die das hier:

Das Standardmodell bricht mit der allgemeinen Kovarianz der GR indem mit der Urknalltheorie eine Interpretation bezorzugt wird.

belegen würde?Der Satz ist äußerst seltsam, dafür wird man wohl keine Belege finden.
Allgemeine Kovarianz ist eine Grundeigenschaft der ART. Wenn "mir ihr brechen" heißen soll, dass sie nicht mehr gilt, dann ist das offensichtlicher Unsinn.
Natürlich ist es so, dass spezielle Lösungen der Feldgleichungen eigentlich immer um bestimmte Symmetrien herumgebaut sind, die man von der Raumzeit fordert. Die Schwarzschildlösung z. B. setzt räumliche Kugelsymmetrie und Vakuum voraus, die RW-Metrik räumliche Homogenität und Isotropie, die Gödel-Metrik räumliche Homogenität ohne Isotropie und so weiter. Entsprechend erlauben auch die Koordinatensysteme, die entlang dieser Symmetrien konstruiert wurden, eine besonders einfache Beschreibung aller Vorgänge, für die diese Symmetrien wichtig sind. In diesem Sinne sind diese Koordinatensystem ausgezeichnet vor anderen.
Das bricht aber beim besten Willen nicht mit der allgemeinen Kovarianz. Sehen wir uns als Beispiel die SRT an mit ihrer speziellen Kovarianz: "die Naturgesetze nehmen dieselbe Form an in allen relativ zueinander bewegten Inertialsystemen". Das ist und bleibt richtig, trotzdem wird die Beschreibung eines Körpers in dessen Ruhesystem die einfachste sein. Ebenso ist es in der ART.

Dann haben wir noch die Aussage, dass die Urknalltheorie eine bevorzugte Interpretation der Raumzeit/des Standardmodells (geht nicht hervor) sei. Das ist nun absoluter Blödsinn. Die Urknalltheorie ergibt sich vollkommen kovariant und interpretationsfrei aus der bekannten Bewegung im heutigen Universum. Da gibt es Singularitätstheoreme, die (mit Einschränkungen) beweisen, dass die jetzigen Bedingungen in der Zeit zurückgerechnet zu einer Singularität, aka Big Bang, führen. Was in der Realität als "heißer, dichter Anfangszustand" zu lesen ist, siehe hier (http://www.einstein-online.info/spotlights/big_bangs). Das ist die Urknalltheorie, und sie hat nichts mit Koordinatensystemen zu tun.
Die kommen erst beim Standardmodell ins Spiel. Dieses ist, wie der Name schon sagt, ein möglichst einfaches mathematisches Modell, das die beobachtete Wirklichkeit in ausreichender Näherung wiedergibt. Es erlaubt quantitative "Vorhersagen" dessen, was wir sehen (obwohl unbefriedigenderweise die Postdiktion dominiert). Das bedeutet, dass die angenommenen Symmetrien zumindest näherungsweise gültig sind - und ihrerseits erklärt werden müssen, was zum Inflationsszenario geführt hat.

Dem Satz ist also schwer Sinn zu entlocken. Ich gehe mal davon aus, dass GP da von der Neigung mancher Kosmologen fehlgeleitet wurde, das mitbewegte Koordinatensystem quasi vorzuschreiben, um bestimmte Missverständnisse zu vermeiden.
Der entscheidende Punkt ist folgender: allgemeine Kovarianz bedeutet, dass alle möglichen geeigneten Koordinatensysteme verwendet werden können, ohne die Physik zu ändern. Mit diesen KS kommen Interpretationen, und auch diese sind nicht richtig oder falsch, sondern brauchbar oder unbrauchbar. Die Kovarianz bedeutet, dass man sie alle verwenden darf, wenn man will.
Wenn hier jemand behauptet hätte, das Auseinanderbewegen von Galaxien sei ausschließlich als Relativbewegung zu interpretieren und ein Modell "expandierenden Raums" sei deswegen ausgeschlossen, dann wäre das falsch. Ebenso falsch ist es, zu behaupten, die Interpretation als expandierender Raum schließe auf allen Skalen eine Interpretation als Relativbewegung aus.
Richtig ist, wie Peacock sagt, dass die kinematische Interpretation bestimmte globale Eigenschaften des expandierenden Universums nicht einzufangen vermag und andererseits die Interpretation als expandierender Raum in der Betrachtung lokaler Physik vollkommen sinnfrei und irreführend wird. Und dass deswegen beide Interpretationen, je nach Fragestellung, zuzulassen sind. Eigentlich eine Binsenweisheit, aber gerade in der Kosmologie kann man damit sehr viele Leute aus dem Konzept bringen.

Timm
04.03.14, 18:54
allgemeine Kovarianz bedeutet, dass alle möglichen geeigneten Koordinatensysteme verwendet werden können, ohne die Physik zu ändern.
Umso erstaunlicher ist es, wenn das Argument der Kovarianz zwar gebracht, aber mit Verweis auf die Intuition gleich wieder über Bord geworfen wird.
Eigentlich heißt das ja, Intuition bricht Physik. Diese Sichtweise ins rechte Licht zu rücken, dürfte das gemeinsame Motiv all dieser Autoren sein, die genannt wurden.

Ich
05.03.14, 08:54
Umso erstaunlicher ist es, wenn das Argument der Kovarianz zwar gebracht, aber mit Verweis auf die Intuition gleich wieder über Bord geworfen wird. Wo wäre das gewesen?
Ich weiß nicht genau, was er wollte und woran er gescheitert ist. Er hat offensichtlich den Leuten Meinungen untergeschoben, die sie nicht vertreten, und sie dann aufgrund dieser Meinungen attackiert, auch persönlich. Mal innezuhalten und verstehen zu wollen, worüber wir hier eigentlich diskutieren hatte er trotz Warnung keine Lust.

Frage an alle: wir sind hier eigentlich durch, oder? Allgemeine Kovarianz sagt, man kann sich die Galaxien im Sinne der Ausgangsfrage des Threads auch bewegt vorstellen, das steht auch so im Lehrbuch, von daher ist das experimentell nicht von Raumexpansion zu unterscheiden. Erst als globale Beschreibung hat die Expansion des Raums Alleinstellungsmerkmale.

Bauhof
05.03.14, 10:13
Allgemeine Kovarianz sagt, man kann sich die Galaxien im Sinne der Ausgangsfrage des Threads auch bewegt vorstellen, das steht auch so im Lehrbuch, von daher ist das experimentell nicht von Raumexpansion zu unterscheiden.

Hallo ICH,

wenn man sich die Galaxien bewegt vorstellen kann, dann habe ich noch nicht verstanden, welches Bezugssystem dafür maßgebend ist. Relativ zu was bewegen sich die Galaxien?

Erst als globale Beschreibung hat die Expansion des Raums Alleinstellungsmerkmale.

Statt "globale Beschreibung" würde ich das Kriterium der Model-Wahl sehen, wie es bereits Hawkwind hier (http://www.quanten.de/forum/showpost.php5?p=74753&postcount=36) angedeutet hat. Beim in sich zurückgeschlossenem Universum-Modell – auch wenn das zur Zeit nicht aktuell ist – kann ich mir keine andere Deutung als die Expansion des Raumes vorstellen.

Nur wenn man sich ein hypothetisches Bezugssystem vorstellt, das außerhalb der drei Dimensionen des Raumes liegt, dann kann man auch bei diesem Modell die Galaxien als bewegt ansehen.

M.f.G. Eugen Bauhof

Ich
05.03.14, 13:04
wenn man sich die Galaxien bewegt vorstellen kann, dann habe ich noch nicht verstanden, welches Bezugssystem dafür maßgebend ist. Relativ zu was bewegen sich die Galaxien? Zueinander. Wie bei Bewegung üblich kannst du dafür beliebige Bezugssysteme angeben. A bewegt sich relativ zu B und B relativ zu A.
Statt "globale Beschreibung" würde ich das Kriterium der Model-Wahl sehen, wie es bereits Hawkwind hier angedeutet hat. Beim in sich zurückgeschlossenem Universum-Modell – auch wenn das zur Zeit nicht aktuell ist – kann ich mir keine andere Deutung als die Expansion des Raumes vorstellen.Das tut nichts zur Sache. Du kannst (und wirst wahrscheinlich) natürlich Raumexpansion als dein Modell wählen, wenn du mit topologischen Aspekten zu tun hast. Wenn du aber dieselbe Deutung bzw. dasselbe Modell wählst, um z. B. das Sonnensystem zu modellieren, bist du selbst schuld. Das wäre einfach Blödsinn.

Das Wichtige ist, und das habe ich schon oft betont: Du musst dich nicht für ein Modell entscheiden, das du dann auschließlich verwendest. Das wäre sogar kontraproduktiv.
Die ART gibt mathematisch die richtige Lösung, unabhängig von den Modellen, die man sich dazudenkt. Damit kann man aber wenig anfangen, man möchte ja auch Intuition entwickeln und die Geschehnisse mit seiner Erfahrungswelt in Zusammenhang bringen. Dafür gibt es diese Modelle. Und je mehr funktionierende Modelle es für einen Sachverhalt gibt, umso besser. Man kann gar nichts schlechteres machen als sich für eines zu entscheiden.

Und für diesen Thread hieß es:
Man hat irgendwo ein Stück Raum, keine störenden Massen in der Nähe und Zeit genug.Da ist es vollkommen egal, welche Topologie das Universum hat. Die Frage war: kann man in diesem Stück Raum Expansion experimentell von Raumausdehnung unterscheiden? Kann man also irgendwie feststellen, ob sich die Körper "wirklich" bewegen oder einer Ausdehnung folgen? Man kann nicht.

Bauhof
05.03.14, 13:45
Das Wichtige ist, und das habe ich schon oft betont: Du musst dich nicht für ein Modell entscheiden, das du dann auschließlich verwendest. Das wäre sogar kontraproduktiv.
Die ART gibt mathematisch die richtige Lösung, unabhängig von den Modellen, die man sich dazudenkt. Damit kann man aber wenig anfangen, man möchte ja auch Intuition entwickeln und die Geschehnisse mit seiner Erfahrungswelt in Zusammenhang bringen. Dafür gibt es diese Modelle. Und je mehr funktionierende Modelle es für einen Sachverhalt gibt, umso besser. Man kann gar nichts schlechteres machen als sich für eines zu entscheiden.

Hallo ICH,

ich habe mal gelesen, dass das ein Manko der ART sei, dass sie praktische alle möglichen Varianten hinsichtlich der Universum-Expansion offen lässt. Damit ist die ART kaum widerlegbar und das ist nicht gut. Eine physikalische Theorie muss widerlegbar sein, je "widerlegbarer" sie ist, desto besser ist sie (Karl Popper).

Anfangs übersah Einstein mit der ART überhaupt die Möglichkeit einer Universum-Expansion. Er glaubte an ein statisches Universum. Vermutlich erst nachdem das "Auseinanderfliegen" der Galaxien anhand der kosmologischen Rotverschiebung beobachtet wurde, führte er sein "kosmologisches Glied" ein. Es kann aber auch sein, dass er das schon vorher einführte, weil ihn Alexander Friedmann auf die Möglichkeit einer Expansion hinwies. Soviel zur Leistung der ART-Mathematik.

M.f.G. Eugen Bauhof

Ich
05.03.14, 15:03
ich habe mal gelesen, dass das ein Manko der ART sei, dass sie praktische alle möglichen Varianten hinsichtlich der Universum-Expansion offen lässt. Damit ist die ART kaum widerlegbar und das ist nicht gut. Eine physikalische Theorie muss widerlegbar sein, je "widerlegbarer" sie ist, desto besser ist sie (Karl Popper).Das ist etwas anderes. Die quantitativen Vorhersagen einer Theorie sind unabhängig von den Modellen, die man benutzt. Das berührt die hier gestellte Frage also nicht.

Es ist so, dass die ART nicht vorgibt, dass das Universum mit einem homogenen, isotropen Weltmodell gerechnet werden kann. Das ergibt sich aus den Beobachtungsdaten. Und selbst das vorausgesetzt, macht die ART über die Dynamik kaum eine Aussage, außer dass es keine stabile statische Lösung gibt. Was ja auch schon mal was ist.
Die ART ist, was Kosmologie angeht, in der Tat ziemlich elastisch. Damit könnte man sehr viele verschiedene Universen modellieren. Deswegen ist sie aber noch lange nicht beliebig. Die Evolution muss ihren Gesetzen folgen, und daraus folgt, dass die eine Beobachtung die möglichen Ergebnisse einer anderen Beobachtung stark einschränkt. Zum Beispiel zeigt die Beobachtung der Hintergrundstrahlung, dass das Universum flach ist, und alleine daraus (nud der heutigen Massendichde und Hupplekonstante) kann entnehmen, dass etwa 70 % der Energiedichte weder baryonisch noch "dunkel" sein kann. Aus den SN1a-Beobachtungen kommt man darauf, dass etwa 70% der Energiedichte dunkle Energie sein müssen. Die beobachteten großräumigen Strukturen im Universum wiederum lassen sich in ihrer Evolution nur erklären, wenn man ca. 70% dunkle Energie annimmt. Würden diese Werte nicht zusammenpassen, wäre das Standardmodell und wahrscheinlich die gesamte ART-Kosmologie falsifiziert.
Er glaubte an ein statisches Universum. Vermutlich erst nachdem das "Auseinanderfliegen" der Galaxien anhand der kosmologischen Rotverschiebung beobachtet wurde, führte er sein "kosmologisches Glied" ein. Es kann aber auch sein, dass er das schon vorher einführte, weil ihn Alexander Friedmann auf die Möglichkeit einer Expansion hinwies. Soviel zur Leistung der ART-Mathematik.
Na, na. Lambda ist schlicht und einfach in der Theorie mit drin. Die Konstante zu Null zu setzen wäre nur die einfachste Möglichkeit gewesen, es gibt dafür aber keinen weiteren triftigen Grund. Es ist übrigens so, dass diese Konstante ein (instabiles) statisches Universum überhaupt ermöglicht, und Einstein sie erst nach der Entdeckung der Expansion herausgeschmissen hat, weil sie nicht mehr nötig war - nach dem richtigen Wahlspruch "so einfach wie möglich". Erst in neuerer Zeit wurde sie wiederbelebt, um dem zweiten Teil des Wahlspruchs gerecht zu werden: "aber nicht einfacher".

Timm
14.03.14, 09:11
Frage an alle: wir sind hier eigentlich durch, oder?
Fast. Man könnte noch mit der Annahme argumentieren, das Universum expandiere zu den Zeitpunkten von Emission und Absorption nicht, dazwischen aber wohl.

Also ist das Argument der Relativbewegung vom Tisch.

Also ist die Rotverschiebung zwingend durch die Dehnung von Raum verursacht.

Also ist die Dehnung von Raum Physik.

Ich
14.03.14, 12:17
Fast. Man könnte noch mit der Annahme argumentieren, das Universum expandiere zu den Zeitpunkten von Emission und Absorption nicht, dazwischen aber wohl.

Also ist das Argument der Relativbewegung vom Tisch.
Kommt drauf an, die Variante mit dem Vergleich entlang des Lichtwegst ist davon unberührt.
Aber die interessantere Definition von Relativbewegung, die entlang raumartiger Geodäten, ist damit zumindest deutlich reduziert. Das macht aber nichts, weil hier ja auch noch gravitative Rotverschiebung eine Rolle spielt. Ein ganzes Universum anzuhalten und wieder zu beschleunigen braucht ziemlich viel Gravitation.
Das müsste ich mal in Näherung rechnen. Mal schauen, wie sich das darstellt.

Timm
14.03.14, 15:31
"Also ist ..." war die Sicht eines advocatus diaboli. :D

Das Szenario wird von Bunn&Hogg (http://arxiv.org/PS_cache/arxiv/pdf/0808/0808.1081v2.pdf) - allerdings ohne diesen
Ein ganzes Universum anzuhalten und wieder zu beschleunigen braucht ziemlich viel Gravitation.
Das müsste ich mal in Näherung rechnen. Mal schauen, wie sich das darstellt.

Einwand - diskutiert.
Was die beiden Orte physikalisch verbindet ist der Pfad des Lichts. Insofern ist finde ich der Paralleltransport der vierer-Geschwindigkeit auf diesem Pfad ein vernünftiger (zumindest nicht falscher) Vorschlag, während es eher problematisch ist, eine Relativgeschwindigkeit an diesen beiden Zeitpunkten (Emission - Absorption) festzumachen. Jedenfalls, wenn ich es richtig verstehe, wären so definierte Relativgeschwindigkeiten von der zeitlichen Entwicklung des Skalenfaktors abgekoppelt.
Aber die interessantere Definition von Relativbewegung, die entlang raumartiger Geodäten, ist damit zumindest deutlich reduziert. Das macht aber nichts, weil hier ja auch noch gravitative Rotverschiebung eine Rolle spielt.
Kannst Du das nochmal etwas ausführlicher darstellen?

Ich
14.03.14, 16:03
Jedenfalls, wenn ich es richtig verstehe, wären so definierte Relativgeschwindigkeiten von der zeitlichen Entwicklung des Skalenfaktors abgekoppelt. Nicht ganz, die Gleichzeitigkeitsdefinition bei meiner Lieblingsrelativgeschwindigkeit ist ja nicht identisch mit der FRW-Definition. Von daher würde dieser Paralleltransport in die FRW-Vergangenheit erfolgen und hängt also grundsätzlich von der Historie der Expansion ab. Die Geodäte würde auch nicht genau die Zeitpunkte treffen, bei denen die Geschwindigkeit Null war. Das ist ein ganz klares Beispiel dafür, dass Geschwindigkeiten entfernte Objekte stark von der Definition abhängen - übrigens auch schon in der SRT.

Aber darum geht es nicht wirklich, sondern wir wollen ja den Sender zum Zeitpunkt der Emission und den Empfänger zum Zeitpunkt der Absorption zueinander in Ruhe haben, was immer das genau bedeutet.
Kannst Du das nochmal etwas ausführlicher darstellen?Ich müsste es erst noch selber rechnen, am besten wohl in der Newtonschen Näherung. Ich weiß noch nicht, wann ich dazu komme. Aber interessieren würde es mich schon, keine Frage.

Timm
14.03.14, 17:50
Aber darum geht es nicht wirklich, sondern wir wollen ja den Sender zum Zeitpunkt der Emission und den Empfänger zum Zeitpunkt der Absorption zueinander in Ruhe haben, was immer das genau bedeutet.
Ich müsste es erst noch selber rechnen, am besten wohl in der Newtonschen Näherung. Ich weiß noch nicht, wann ich dazu komme. Aber interessieren würde es mich schon, keine Frage.
Und da bin ich ziemlich sicher, nicht nur auch mich.

Timm
18.04.14, 09:29
Es ist natürlich kein Wunder, daß diese Diskussion immer wieder da und dort geführt wird.

Wen's interessiert, etwa ab hier (http://www.physicsforums.com/showpost.php?p=4719550&postcount=76) ff. wird's spannend.
... can anyone cite evidence that is happening? Of course not, we have no model of space, we have only the predictions of GR. No part of the mathematics of GR says space is expanding, it's pure made-up language, accepted uncritically as if it was really saying something we could ever test, which then prompts people to ask "where does the space come from." That's my point here, the question emerges from a non-model, no part of GR asserts that space is actually expanding. Indeed, it seems to me one of the main points of all of relativity is noticing the difference between observations and coordinate systems!
Meine z.T. etwas freie Übersetzung:

... Gibt es einen Nachweis, daß das passiert (er meint die Expansion des Raums). Natürlich nicht, wir haben kein Modell vom Raum, wir haben nur die Vorhersagen der ART. Nirgendwo folgt aus der Mathematik der ART, daß Raum expandiert, das ist eine reine Sprachregelung, die nahelegt, man könnte das testen und die dann Leute auf die Frage bringt "Woher kommt der Raum". ... diese Frage beruht nicht auf einem Modell, nicht auf der ART. Mir scheint, einer der wesentlichen Punkte ist es, den Unterschied zwischen Beobachtungen und Koordinatensystemen zu erkennen!

Aus Post #64 desselben Threads:

Until Penrose and the twistor program came along, it was necessary to use what are called field equations in order to determine a field's behaviour. But today, with the help of the rich cohomology of twistor space. It becomes possible to get rid of the differential equations that determine the field. The twistor picture relies purely on the geometrical (or cohomological) properties of the field as it it expressed in terms of twistors and twistor space. This is a truly amazing result, for it means that the twistor approach can deal with the various fields of nature without ever needing to bother about field equations!
Your question, therefore, is mute in reference to twistor space, because the picture is radically different in Penrose's twistor approach, for the massless fields are now defined in (projective) twistor space. But since this space has only three complex dimensions, it turns out that the information about the field's structure will totally fill twistor space!
There is no room left in the twistor picture, nothing else for the field to do, no additional slice of space to fill...and, because twistor space is totally filled with the field's structure, there is no need for a field equation...the field along with all its dynamics is already totally defined, fully represented within the twistor picture.
Falls das so ist, daß die Feldgleichungen verzichtbar sind, weil Penrose's twistor space sie implizit enthält (wenn ich es richtig verstanden habe), dann wäre das doch eine sehr beachtliche Erkenntnis. Andererseits scheint die LQG aber nicht recht voran zu kommen. Ok, das jetzt ein anderes Thema, passt nicht hier her.

Ich
10.05.14, 21:41
Ich müsste es erst noch selber rechnen, am besten wohl in der Newtonschen Näherung. Ich weiß noch nicht, wann ich dazu komme.
...und schon ist es so weit. Ich habe die Sache durchgerechnet, mit dem erwarteten Ergebnis. Die Mathematik ist ganz nett: partielle Integration, die schon in der Schulformelsammlung steht (wenn ich mich richtig erinnere) gibt genau die Beziehung zwischen den verschiedenen Arten von Rotverschiebung. Deswegen schicke ich ein File mit der Rechnung mit, hier im Forum funktioniert das mangels Tex ja leider nicht.
Also, falls es tatsächlich noch dich, Timm, und andere interessiert:
Rotverschiebung (http://www.file-upload.net/download-8905008/Rotverschiebung.pdf.html)

Marco Polo
11.05.14, 02:19
Beim Download der pdf-Datei habe ich mir einen Trojaner eingefangen.

Zumindest behauptet das mein Antivirus Programm, das diesen hoffentlich erfolgreich eliminiert hat.

Kannst du die Berechnung nicht einfach als Anhang hier einstellen?

Ich
11.05.14, 09:53
Beim Download der pdf-Datei habe ich mir einen Trojaner eingefangen.
Echt? Die Seite war grün im WOT, aber das muss wohl nichts heißen. Sorry.

Ich hab das gar nicht geblickt, dass man hier Anhänge reinstellen kann. Die Datei ist aber zu groß, ich muss sie teilen.

Timm
11.05.14, 10:08
Also, falls es tatsächlich noch dich, Timm, und andere interessiert:
Rotverschiebung (http://www.file-upload.net/download-8905008/Rotverschiebung.pdf.html)
Natürlich bin ich da sehr gespannt, und andere bestimmt auch. Bin aber jetzt nach Marc's Erfahrung erst mal vorsichtig. Vielleicht findest Du eine andere Möglichkeit.
Eigentlich ist Tex oder etwas vergleichbares kein übertriebener Luxus für ein Physikforum. Ich erinnere mich im Moment nicht, daß wir das Thema schon mal mit Günter hatten und kann auch den Aufwand nicht beurteilen. Man könnte ihn ja mal fragen.

Marco Polo
11.05.14, 10:44
Eigentlich ist Tex oder etwas vergleichbares kein übertriebener Luxus für ein Physikforum. Ich erinnere mich im Moment nicht, daß wir das Thema schon mal mit Günter hatten und kann auch den Aufwand nicht beurteilen. Man könnte ihn ja mal fragen.

Das Thema habe ich bereits mehrfach mit unserem Admin besprochen.

Leider bisher ohne Ergebnis. Es müsste dazu auf die neue Version der Forensoftware umgestellt werden heisst es. Scheinbar bedeutet das zuviel Aufwand.

Vielleicht sollte man aber auch nicht immer nur fordern, sondern sich auch mal mit dem zufrieden geben, was man zur Verfügung gestellt bekommt.

Das Betreiben eines Forums ist mit Zeit und Kosten verbunden und wir sollten unserem Admin dankbar sein, dass er uns diese Plattform zur Verfügung stellt.

Trotzdem wurmt es mich natürlich, dass es hier kein Tex gibt. :(

Marco Polo
11.05.14, 10:55
Echt? Die Seite war grün im WOT, aber das muss wohl nichts heißen. Sorry.

Ich hab das gar nicht geblickt, dass man hier Anhänge reinstellen kann. Die Datei ist aber zu groß, ich muss sie teilen.

Danke an "Ich" :)

Funktioniert. Interessante Berechnung in einem professionellen Layout (sieht nach LaTex aus).

Timm
11.05.14, 18:35
Die Datei ist aber zu groß, ich muss sie teilen.
Danke, ist jetzt erst mal nur ausgedruckt.

Timm
16.05.14, 10:01
Wenn ich sie richtig verstehe, baut sich Deine Herleitung der kosmologischen Rotverschiebung auf der zeitlichen Entwicklung des Skalenfaktors X(t) auf, kein Paralleltransport eines Vektors, keine Akkumulation infinitesimaler Dopplerverschiebungen, kein Gravitationspotential (Peacock), nichts was mir schon mal untergekommen wäre. Und bei einer hypothetischen linearen Expansion würde wegen a(t) = 0 der gravitative Anteil erwartungsgemäß verschwinden.
Könntest Du v(t) in (2) etwas ausführlicher erläutern, was genau bewegt sich relativ wozu. Ich sehe zwar die Proportionalität zu X(t), bin aber nicht sicher, wie ich das zu deuten habe.

Bauhof
17.05.14, 17:36
Ich hab das gar nicht geblickt, dass man hier Anhänge reinstellen kann. Die Datei ist aber zu groß, ich muss sie teilen.

Hallo ICH,

in deinem PDF hast du geschrieben:

Die in (3) angegebene Beschleunigung ist rein gravitativer Natur, weil mitbewegte Beobachter kräftefrei sind – jede Relativbeschleunigung ist also auf eine Scheinkraft, die Gravitationskraft, zurückzuführen. Da die Beschleunigung anfänglich positiv sein soll, ist das zugehörige Gravitationsfeld vom Ursprung weg gerichtet und führt daher zu einer Rotverschiebung des zum Ursprung wandernden Signals.

Du sprichst von einem "zugehörigen Gravitationsfeld". Wo ist die Quelle dieses Gravitationsfeldes? Wo ist die zugehörige gravitierende Masse?

M.f.G. Eugen Bauhof

P.S.
Nun habe ich jahrelang meinen Teil dazu beigetragen, dass die Trolle und Spinner hier nach und nach eliminiert wurden. Aber dadurch hat sich das Beitrags-Niveau (u. a. durch dich) so erhöht, dass ich kaum mehr mitreden kann. :rolleyes:

Ich
17.05.14, 22:10
Hallo Timm,
Wenn ich sie richtig verstehe, baut sich Deine Herleitung der kosmologischen Rotverschiebung auf der zeitlichen Entwicklung des Skalenfaktors X(t) auf, kein Paralleltransport eines Vektors, keine Akkumulation infinitesimaler Dopplerverschiebungen, kein Gravitationspotential (Peacock), nichts was mir schon mal untergekommen wäre.
Doch, das sind schon bekannte Zutaten. Der Zusatzterm in (14) ist ganz einfach die Dopplerverschiebung durch die Relativbewegung von Quelle und Empfänger zum Sendezeitpunkt. Würde ich hier mit Paralleltransport arbeiten wie Bunn&Hogg, würde die Raumzeitkrümmung (aka Gravitation) da mit eingehen und es gäbe keine weiteren Terme. Stattdessen arbeite ich vor einem flachen Hintergrund - das ist die übliche Näherung für schwache Felder. Eigentlich sogar Newtonsche Näherung, weil auch alle Relativgeschwindigkeiten als klein angenommen werden. Die Gravitation taucht dann explizit auf.
Das tut sie in (5). a(t,x)*dx (aus dem Schulunterricht bekannt als "g*h") ist das Gravitationspotential, das das Licht überwindet, wenn es um dx näher kommt. Die Rotverschiebung ist der Potentialunterschied durch c², also (5). Das Integral (7) ist dann die gravitative Rotverschiebung zwischen Sender und Empfänger.

Und bei einer hypothetischen linearen Expansion würde wegen a(t) = 0 der gravitative Anteil erwartungsgemäß verschwinden. Richtig. Dann könnte die Expansion aber auch nicht beginnen und wieder enden, wie in deinem Beispiel vorgesehen.

Könntest Du v(t) in (2) etwas ausführlicher erläutern, was genau bewegt sich relativ wozu. Ich sehe zwar die Proportionalität zu X(t), bin aber nicht sicher, wie ich das zu deuten habe.Klein v(t) ist die Relativgeschwindigkeit der Quelle zum Empfänger. Gemäß der Definition aus (1) ist das die "recession velocity", und (2) ist ganz einfach das Hubblesche Gesetz: Groß V ist der Hubbleparameter, x0 die mitbewegte Entfernung.

Ich
17.05.14, 22:38
Hallo Eugen,

Du sprichst von einem "zugehörigen Gravitationsfeld". Wo ist die Quelle dieses Gravitationsfeldes? Wo ist die zugehörige gravitierende Masse?

Das "zugehörig" bezieht sich erst mal auf die Beschleunigung: Ein Beschleunigungssensor würde für jeden mitbewegten Beobachter bestätigen, dass er unbeschleunigt ist, dennoch gibt es eine Relativbeschleunigung zu allen anderen. Wollte man diese Relativbeschleunigung durch eine Kraft erklären, dann erfüllt diese die Definition einer Scheinkraft. In unserem Fall ist das Gravitation.
Deren Quelle ist hier nicht näher definiert, weil sich das pdf nur mit den Auswirkungen beschäftigt. In Timms Beispiel (Ruhe - Beschleunigung - Abbremsen - Ruhe) fällt mir auch keine sonderlich plausible Quelle ein. Ein anfänglich inflationäres Universum, das dann wieder zusammenfällt, würde passen.
Die Quelle der Gravitation ist immer die Materie/Energie/Druck, die das Universum erfüllen. Gravitation tritt dabei als relative Beschleunigung auf, also jedes Teilchen wird zu jedem anderen hin (oder weg) beschleunigt, proportional zur Entfernung.
Wenn du dir irgendein lokales Inertialsystem aussuchst und darin die Dinge in Newtonscher Näherung betrachtest, dann stellt sich das folgendermaßen da:
Nimm einen Punkt in Entfernung x vom Ursprung. Das Universum ist um den Ursprung (und um jeden anderen Punkt) kugelsymmetrisch. Nach dem Schalentheorem (dem von mir vielzitierte Birkhoff in der ART) reicht es dann, eine Kugel mit Radius x um den Ursprung zu betrachten: der kugelsymmetrische Außenraum übt keine Kraft aus. Innerhalb dieser Kugel befindet sich eine bestimmte Menge Zeugs. Und diese Zeugs zieht das Teilchen an, Richtung Ursprung.
Also, als Antwort auf deine Frage: Nimm zwei beliebige Punkte, sie werden durch Gravitation relativ zueinander beschleunigen. Die Quelle dieser Gravitation ist die Materie zwischen diesen Punkten. Das Ziehen der Materíe außerhalb hebt sich weg.
Nun habe ich jahrelang meinen Teil dazu beigetragen, dass die Trolle und Spinner hier nach und nach eliminiert wurden. Aber dadurch hat sich das Beitrags-Niveau (u. a. durch dich) so erhöht, dass ich kaum mehr mitreden kann. :rolleyes:Danke für das Lob bezüglich des Niveaus. Quantitativ hat sich hier allerdings einiges reduziert. Ich würde mich freuen, wenn mehr normale Leute hier aufschlagen und mitdiskutieren würden. Die Spinner brauch' ich auch nicht zurück.

Timm
18.05.14, 10:26
Du sprichst von einem "zugehörigen Gravitationsfeld". Wo ist die Quelle dieses Gravitationsfeldes? Wo ist die zugehörige gravitierende Masse?

Hallo Eugen,

ich kann diese Fragen gut nachempfinden, denn ich habe sie mir auch gestellt. Das Problem ist, man kann sich die Wirkung der Gravitation im Fall einer zentralen Masse (Schwarzschild Modell) im Raum noch ganz gut vorstellen, aber nicht mehr - ich jedenfalls nicht - wenn die Masse den Raum homogen und isotrop füllt (FRW Modell).

Eine wesentlich Wirkung der Gravitation ist die Gezeitenbeschleunigung. Fallen 2 Objekte radial hintereinander in Richtung der Masse, so beschleunigen sie dabei voneinander weg. Das kann man sich qualitativ ohne eine Formel noch klarmachen. Aber die Intuition hat wohl keine Chance im Fall des FRW Modells mit der homogenen Masse ohne Zentrum.
Dann bleibt einem nichts anderes übrig, als der Friedmann'schen Bescheunigungsgleichung zu vertrauen. Nach ihr ist die relative Beschleunigung von mitbewegten Objekten proportional zu - (Energiedichte + 3*Druck). Und man sieht auf einen Blick, daß sie aufeinander zu beschleunigen, wenn der Druck positiv (oder null) ist und voneinander weg, wenn der Klammerausdruck negativ ist. Was nur mit hinreichend großem negativen Druck (kosmologische Konstante) möglich ist.
Ich kann Dir bestimmt nicht viel Neues sagen, wollte aber wenigstens kundtun, wie ich mit derselben Problematik umgehe.

Gruß, Timm

P.S. Hoffentlich habe ich nicht Verwirrung gestiftet. denn Ich hat Deine Frage aus der Sicht der Newtonschen Näherung beantwortet.

Ich
18.05.14, 15:21
Aber die Intuition hat wohl keine Chance im Fall des FRW Modells mit der homogenen Masse ohne Zentrum.Mir haben die folgenden zwei Schritte geholfen, wenn man in Newtonscher Gravitation denkt:
1. Nur eine Kugel betrachten, kein unendliches Universum. Die fängt an zu kollabieren. Das kann man sich gut vorstellen und rechnen.
2. Dann innerhalb dieser Kugel eine kleinere, außermittige Kugel betrachten. Deren Beschleunigung lässt sich in zwei Komponenten aufteilen:
a) Die Kugel als Ganzes fällt zum Zentrum.
b) Die Kugel selbst kollabiert ebenfalls, genau wie die große Kugel.

a) ist innerhalb der kleinen Kugel nicht feststellbar. Das ist wie ein homogenes Gravitationsfeld, das man einfach wegtransformieren kann. Was bleibt ist b): jede Kugel sieht sich selbst kollabieren, nicht jedoch irgendwohin beschleunigen.

Wenn man jetzt die große Kugel unendlich groß werden lässt, ändert sich nichts Prinzipielles: jede beliebige Kugel darin kollabiert auf die gleiche Art. Dass sie bezüglich irgendeines unendlich fernen Punktes unendlich stark beschleunigt macht nichts, weil das nicht feststellbar ist.

Bauhof
18.05.14, 15:32
Hallo Eugen,
ich kann diese Fragen gut nachempfinden, denn ich habe sie mir auch gestellt. Das Problem ist, man kann sich die Wirkung der Gravitation im Fall einer zentralen Masse (Schwarzschild Modell) im Raum noch ganz gut vorstellen, aber nicht mehr – ich jedenfalls nicht – wenn die Masse den Raum homogen und isotrop füllt (FRW Modell).

Hallo Timm,

das kann ich mir auch nicht vorstellen. Noch weniger kann ich mir den Zusammenhang zwischen der Gravitation und der Universum-Expansion bei folgendem Weltmodell vorstellen:

Unser dreidimensionaler Weltraum ist der Begrenzungsraum eines vierdimensionalen Objekts, im einfachsten Fall einer vierdimensionalen Kugel. So wie es etwa Stephen Hawking in seinem Buch "Eine kurze Geschichte der Zeit" darstellt.

Der vierdimensionale Radius dieser Kugel wächst konform mit der Universum-Expansion und damit vergrößert sich auch deren Begrenzungsraum, unser Weltraum. In welcher Weise können die im 3-D-Raum vorhandenen Massen Einfluss auf das Größerwerden des 4-D-Radius nehmen? Abbremsend? Beschleunigend? Wie soll das gehen? Alleine mit Mathematik kann man da nichts gewinnen.

Ich kann Dir bestimmt nicht viel Neues sagen, wollte aber wenigstens kundtun, wie ich mit derselben Problematik umgehe.

Doch, du sagst mir Neues, denn mein kosmologisches Wissen ist nicht so tiefgreifend wie deines.

M.f.G. Eugen Bauhof

Timm
18.05.14, 16:38
Doch, das sind schon bekannte Zutaten.
Ok.

Aber insgesamt haperts noch am Verständnis mit dem Umgang mit Birkhoff's Theorem um Deinen Ansatz wenigstens vom Grundsatz her nachvollziehen zu können. Deshalb erst nochmal Gl (16) in Peacock's Diatribe. Mit Sender am Rand und Empfänger im Zentrum ergibt sich für
v = 0 : z_kosm ~ - rho r², also eine gravitative Blauverschiebung und für
rho = 0 ist z_kosm die relativistische Dopplerverschiebung.

Da nach diesem Theorem die sphärische Massenverteilung wie eine Punktmasse wirkt, ist die grav. Blauverschiebung keine Überraschung.

Bei Deinem Ansatz sind Sender und Empfänger analog zu Peacock's positioniert. Nach Deiner Gl (14) sind z_grav und z_kin additiv, während bei Peacock auch ein Produkt da steht; z_grav in (14) ist positiv, also Rotverschiebung. Was verstehe ich nun falsch? Eigentlich kann mit "Ursprung" nicht das Zentrum der sphärischen Massenverteilung gemeint sein.

Die Schritte ab (8) verstehe ich nicht, es fehlen einfach die Grundlagen. Du kommst ja - wenn auch anders - wie Peacock zu einer Kombination von gravitativem und kinematischen Effekt. Und wenn ich es richtig verstehe ebenfalls auf Basis Birkhoff. Vielleicht wird manches klarer, wenn Du den wesentlichen Unterschied zu Peacock erläuterst.
Gerade sehe ich was. Behandelst Du das BT dynamisch, Peacock aber statisch?

Timm
18.05.14, 17:22
Hallo Ich,

Mir haben die folgenden zwei Schritte geholfen, wenn man in Newtonscher Gravitation denkt:
1. Nur eine Kugel betrachten, kein unendliches Universum. Die fängt an zu kollabieren. Das kann man sich gut vorstellen und rechnen.

Dazu eine Frage vorab: Wie sieht ein beliebiger innerhalb der kollabierenden Kugel mitbewegter Beobachter die Abstände in seiner Umgebung? Müßten sie nicht radial und transversal schrumpfen, also isotrop?

Marco Polo
18.05.14, 19:51
Dazu eine Frage vorab: Wie sieht ein beliebiger innerhalb der kollabierenden Kugel mitbewegter Beobachter die Abstände in seiner Umgebung? Müßten sie nicht radial und transversal schrumpfen, also isotrop?

Rein prinzipiell bzw. rechnerisch schon, würde ich sagen. Aber die gedachte aussenmittige Kugel wäre ja im Verhältnis zum Universum derart mickrig, dass veränderliche Abstände in dieser (analog zur Kollabierung des Universums) unmessbar klein wären.

Das ist jetzt aber nur aus dem Bauch heraus geschossen. :)

Marco Polo
19.05.14, 00:31
Quantitativ hat sich hier allerdings einiges reduziert. Ich würde mich freuen, wenn mehr normale Leute hier aufschlagen und mitdiskutieren würden.

Ja, das wäre echt wünschenswert. Physik scheint aber leider nicht gerade das Thema zu sein, das die Massen verzückt und uns scharenweisen Zulauf beschert .

Die Spinner brauch' ich auch nicht zurück.

Die braucht keiner.

pauli
19.05.14, 08:15
Ja, das wäre echt wünschenswert. Physik scheint aber leider nicht gerade das Thema zu sein, das die Massen verzückt und uns scharenweisen Zulauf beschert .

leider ... als Laie habe ich in meinem Bekanntenkreis keinen Einzigen, der sich dafür interessiert und mit dem ich über sowas reden kann. Ohne Internet wäre man vollkommen aufgeschmissen.

Timm
19.05.14, 08:38
Ja, das wäre echt wünschenswert. Physik scheint aber leider nicht gerade das Thema zu sein, das die Massen verzückt und uns scharenweisen Zulauf beschert .

Vielleicht braucht man auch etwas Geduld. Es gibt ruhige Phasen und dann wird plötzlich wieder heftig diskutiert, das kennen wir ja. Hier sind Physiker mit den Schwerpunkten Teilchenphysik und RT / Kosmologie aktiv und helfen. Optimale Voraussetzungen interessierte Leute anzuziehen und für Geschwurbel weniger attraktiv zu sein.

Ich
19.05.14, 09:16
Deshalb erst nochmal Gl (16) in Peacock's Diatribe. Mit Sender am Rand und Empfänger im Zentrum ergibt sich für
v = 0 : z_kosm ~ - rho r², also eine gravitative Blauverschiebung und für
rho = 0 ist z_kosm die relativistische Dopplerverschiebung.

Richtig.
Nach Deiner Gl (14) sind z_grav und z_kin additiv, während bei Peacock auch ein Produkt da stehtDer exakte Ausdruck ist:
1+z(gesamt) = (1+z1)*(1+z2), wie bei Peacock.
Weil ich nur in erster Näherung arbeite, schreibe ich dafür
z(zesamt) = z1 + z2, das passt dann auch.
z_grav in (14) ist positiv, also Rotverschiebung. Was verstehe ich nun falsch?Nichts. Du wolltest ja ein Universum, das erst ruht und dann beschleunigt. Dafür muss irgendwas wie Dunkle Energie vorhanden sein, das nach außen gerichtete Gravitation erzeugt.
Eigentlich kann mit "Ursprung" nicht das Zentrum der sphärischen Massenverteilung gemeint sein.Doch, aber nach Aufgabenstellung überwiegt die Abstoßung über die Anziehung.
Die Schritte ab (8) verstehe ich nicht, es fehlen einfach die Grundlagen.(8) hat überhaupt nichts mit 1-7 zu tun. Hier führe ich nur partielle Integration (http://de.wikipedia.org/wiki/Partielle_Integration#Herleitung) ein. (8) ist die Produktregel bei der Ableitung, und (9) ist dasselbe, nur beide Seiten integriert. Erst in (10) setze ich die Funktionen so ein, dass in der Gleichung die beiden Rotverschiebungsterme (4) und (7) auftauchen. Das tun sie dann explizit in (12).
Vielleicht wird manches klarer, wenn Du den wesentlichen Unterschied zu Peacock erläuterst.
Gerade sehe ich was. Behandelst Du das BT dynamisch, Peacock aber statisch?Ich behandle das ganze Problem dynamisch. Peacock gibt einfach das Potential zu einem bestimmten Zeitpunkt an. Deine Fragestellung ist aber dynamisch, das Resultat kommt dadurch zustande, dass sich das Gravitationsfeld während der Lichtlaufzeit ändert. Ich kann also nicht einfach ein statisches Potential einsetzen, sondern muss lauter kleine Potentialunterschiede aufintegrieren, die das Signal überwindet, wenn es um dx weiter kommt. Diese Potentialunterschiede sind das jeweils wirkende Gravitationsfeld a(x,t) multipliziert mit der Strecke dx.
Weil ich a(x,t) nicht festlegen will, sondern die Ableitung für alle möglichen Varianten gelten soll, muss ich das Integral einfach so stehen lassen und damit weiterarbeiten, anstatt es irgendwie auszurechnen.

Ich
19.05.14, 09:55
Dazu eine Frage vorab: Wie sieht ein beliebiger innerhalb der kollabierenden Kugel mitbewegter Beobachter die Abstände in seiner Umgebung? Müßten sie nicht radial und transversal schrumpfen, also isotrop?Da wäre der Aha-Effekt vielleicht am größten, wenn du das ausrechnest. Wenn du Lust dazu hast, gebe ich dir mal die Schritte an:

1. Wie groß ist die Gravitationsbeschleunigung in einem Punkt (x,y,z) innerhalb der Kugel?
Antwort: a=k*(x,y,z). k ist normalerweise negativ, der Wert hängt von der Dichte des Materials ab. Das ist isotrop.
2. Ich mache neue Koordinaten, indem ich oBdA den Ursprung entlang der x-Achse verschiebe: x'=x-x0, y'=y, z'=z. (x0,0,0) soll der Mittelpunkt besagter kleineren Kugel sein.
3. Nach dem Äquivalenzprinzip darf ich ein homogenes Gravitationsfeld dazu addieren, ohne dass sich beobachtbare Resultate ändern: es macht keinen unterschied, ob die ganze Konstruktion frei fällt oder in Ruhe schwebt. Als Zusatzfeld nehme, passend zu meiner Trafo, azusatz = k*(-x0,0,0). Also a'=a+azusatz.
4. Und jetzt müsste rauskommen, dass auch im neuen Koordinatensystem gilt
a'=k*(x',y',z').

Das heißt, egal wo ich innerhalb der Kugel den Ursprung hinlege, es kontrahiert alles homogen und isotrop auf den Ursprung zu. Die isotrope Beschleunigung auf den alten Ursprung zu lässt sich also immer zerlegen in eine isotrope Beschleunigung auf den neuen Ursprung zu und eine (unbeobachtbare) homogene Beschleunigung der ganzen Kugel.

Timm
19.05.14, 18:01
Da wäre der Aha-Effekt vielleicht am größten, wenn du das ausrechnest.
Ja, mit a'=a+azusatz hat man's eigentlich schon. :D
a'=k*(x,y,z)+k*(-x0,0,0) ergibt mit der Verschiebung x'=x-x0 dann
a'=k*(x',y',z')


Das heißt, egal wo ich innerhalb der Kugel den Ursprung hinlege, es kontrahiert alles homogen und isotrop auf den Ursprung zu. Die isotrope Beschleunigung auf den alten Ursprung zu lässt sich also immer zerlegen in eine isotrope Beschleunigung auf den neuen Ursprung zu und eine (unbeobachtbare) homogene Beschleunigung der ganzen Kugel.
Und wenn man abstoßende Gravitation annimmt, gilt das sinngemäß wohl auch.

Was die Dynamik anbelangt, spielt bei der ARTdie Größe des Universums (endlich vs unendlich) keine Rolle. Wie ist das mit der Newton'schen Näherung, kollabiert eine mit unendlichem Radius angenommene Kugel auch?

Ich
19.05.14, 21:07
Und wenn man abstoßende Gravitation annimmt, gilt das sinngemäß wohl auch.Klar, k kann auch positiv sein.
Was die Dynamik anbelangt, spielt bei der ARTdie Größe des Universums (endlich vs unendlich) keine Rolle. Wie ist das mit der Newton'schen Näherung, kollabiert eine mit unendlichem Radius angenommene Kugel auch?Der Radius der großen Kugel spielt keine Rolle, der kommt in den messbaren Größen gar nicht vor.
Das Schalentheorem gilt immer, wenn die Situation kugelsymmetrisch ist, ob unendlich oder nicht. Es ist auch sehr robust: Abweichungen von der Kugelsymmetrie machen nicht plötzlich alle Folgerungen kaputt.
Wenn man sich die Poisson-Gleichung (http://de.wikipedia.org/wiki/Poisson-Gleichung#Gravitation) anschaut, dann sieht man: die mittlere Beschleunigung auf der Oberfläche eines irgendwie abgegrenzten Volumens ist nur abhängig von dem Material in diesem Volumen (Gaußscher Satz). Alles, was außerhalb liegt, kann die genaue Verteilung ändern, wo was wie beschleunigt. Aber ob es in Summe nach innen oder außen geht, hat mit der Außenwelt nichts zu tun.

Timm
20.05.14, 08:39
Das Schalentheorem gilt immer, wenn die Situation kugelsymmetrisch ist, ob unendlich oder nicht.
Also dieser Ausflug zum Schalentheorem/Birkhofftheorem war sehr aufschlussreich, inbesondere die Überlegung mit der kleinen Kugel innerhalb der großen. So lernt man auf seine alten Tage nochwas dazu. :)